Civ Pro Quiz 1

Réussis tes devoirs et examens dès maintenant avec Quizwiz!

How do courts establish Statutory personal jurisdiction over a defendant?

(1) Be present in the forum when served so long as the defendant was not lured fraudulently into the state to participate in a judicial proceeding. (2) domiciled there, (3) consent to jurisdiction (driving through state) or (4) Contacts with forum state.

When is a corporation "at home" for purposes of general jurisdiction?

According to the Court, a corporation is considered to be "at home" wherever it has its principal place of business or place of incorporation.

•A plaintiff sued a defendant corporation for patent infringement in federal district court in State A. The defendant was incorporated in State A but maintained only a mailing address there. The defendant's physical offices, manufacturing plants, and all of its officers, directors, and employees were located in State B. The defendant sold its allegedly infringing product nationwide. •Does the district court in State A have a basis for exercising in personam jurisdiction over the defendant? • •A Yes, because the defendant sells products nationwide. •B Yes, because the defendant is incorporated in State A. •C No, because the defendant is not at home in State A. •D No, because the defendant's principal place of business is in State B.

B

Daenerys from GA sues Viserys from SC for defamation and sexual assault. She seeks $100,000 in damages. In the same lawsuit, she sues Drogo from NC, for battery and intentional infliction of emotional distress for holding her down while she was being assaulted by Viserys. She is seeking $50,000 from him. Assume that Drogo is a necessary and indispensable party under Fed. R. C. P 19. Discuss whether the federal court has subject matter jurisdiction over the entire lawsuit.

First rule: is there diversity? Yes Second rule: Is there supplemental jurisdiction under the second cause of action? Yes, same activity/occurrence Now look to 1367b: if the action is brought by the P against a party added under rule 14, 19, 20, or 24 there is no supplemental jurisdiction. Since there is no supplemental jurisdiction must look to see if there is diversity. There is not because the AIC is not met for the 2nd claim.

28 U.S.C. § 1332 (a)

Diversity of citizenship. District courts shall have original jurisdiction of all civil actions where the AIC is over $75,000, exclusive of interest and cost, AND the parties are citizens of different states

What is Stream of Commerce?

a principle that permits a state to exercise personal jurisdiction over a defendant if the defendant places a product in the general marketplace and the product causes injury or damage in the forum state, provided the defendant also takes other acts to establish some connection with the forum state, e.g., by advertising there or by hiring someone to serve as a sales agent there

What is expressed consent?

Voluntarily going into court or signing a contract with a forum clause.

What are the three types of personal jurisdiction?

1. In personam jurisdiction 2. In rem jurisdiction (meaning power over property, ex. dividing property up during divorce) 3. Quasi in rem jurisdiction (meaning property is not in dispute, but the court is using the defendant's property to try to get jurisdiction. Ex. having property in GA and GA government using that against the defendant, possibly by holding it, to get defendant into court).

What are the 5 factors in the reasonableness of jurisdiction?

1. Plaintiff's interests in obtaining relief. 2. Burden on the defendant. 3. State's interest in adjudicating the dispute. 4. Shared efficiency/efficient resolution of controversies. 5. Interest in enforcement of shared norms.

How do courts establish Constitutional personal jurisdiction over a defendant?

1.) Living in the jurisdiction 2.) Bulge Rule: If you are within 100 miles of the courthouse. 3.) Any person who is in the personal jurisdiction of a local court, is also subject to personal jurisdiction of the federal court. 4.) When authorized by statute.

Emmanuel Macron, aFrench citizen moves to Orlando Florida, to study law. While there, he falls in love with Zooey Deschanel, a dashing brunnete from Florida. They get married while in law school. After finishing law school, they both move to France planning to live there indefinitely. After arriving in France, they both file suitagainst Barrylaw school, in the U.S. District Court for the Middle District ofFlorida, alleging thatatthe end of their first yearin Barry, they showed large reductions in positive affect, life satisfaction, and overall well-being, and large increases in negative affect, depression, and physical symptoms.Assume that they have a legal claim and that the amount in controversy is met. Astotheir lawsuit A. The court lacks jurisdiction, because Zooey is not a citizen of a state or a foreign citizen B.The court lacks jurisdiction, because Zooey remains a citizen of Florida. C.The court has jurisdiction , since neither Zooey nor Macron is domiciled in Florida. D. The court has jurisdiction, because Zooey and Macron have become foreign citizens. Thus the case is between a state corporation, Barrylaw, and citizens of a foreign state.

A

What is a summary judgment?

Motion to ask for dismissal if discovery reveals that a party does not have evidence to establish a fast that is essential to recovery.

A plaintiff sued a defendant corporation for patent infringement in federal district court in State A. The defendant was incorporated in State A but maintained only a mailing address there. The defendant's physical offices, manufacturing plants, and all of its officers, directors, and employees were located in State B. The defendant sold its allegedly infringing product nationwide. Does the district court in State A have a basis for exercising in personam jurisdiction over the defendant? A Yes, because the defendant sells products nationwide. B Yes, because the defendant is incorporated in State A. C No, because the defendant is not at home in State A. D No, because the defendant's principal place of business is in State B.

•Answer option B is correct. A defendant is subject to the personal jurisdiction of his home state. A corporation defendant's home states are at least its state of incorporation and the state where it conducts its principal operations. BNSF Ry. Co. v. Tyrell, 137 S. Ct. 1549, 1558-59 (2017). A corporate defendant may be incorporated in one state and conduct its principal operations in another state, such that, for purposes of personal jurisdiction, the corporate defendant may have two or more home states. See Goodyear Dunlop Tires Operations, S.A. v. Brown, 564 U.S. 915, 924 (2011); Judge Virginia A. Phillips & Judge Karen L. Stevenson, Rutter Group Prac. Guide Fed. Civ. Pro. Before Trial § 3:61 (West April 2019). Here, the defendant corporation is incorporated in State A. The defendant also conducts its principal business in State B, so it has at least two home states: State A and State B. Because State A, the forum state, is one of the defendant's home states, a federal district court in State A has a basis for personal jurisdiction over the defendant. Answer option C is necessarily incorrect for these same reasons.

A citizen of State A purchased life insurance by mail from a State B insurance company. The policy was the only one that the company had ever sold in State A. The purchaser mailed premiums from State A to State B for five years, and then died. The insurance company refused to pay the policy benefits. The purchaser's administrator sued the company in State A state court. The state has a long arm statute that grants a state court in personam jurisdiction over a defendant who "contract[s] to insure any person, property, or risk located within this State at the time of the contracting." The insurance company argued that its only contact with State A since it began its business was the purchaser's insurance policy, and that this single contact does not meet the minimum required for the exercise of in personam jurisdiction under International Shoe. How should the court rule on the minimum contacts issue? A For the purchaser's administrator, because the State A statute authorizes jurisdiction. B For the purchaser's administrator, because of the close connection between the contact and the case. C For the insurance company, because the exercise of jurisdiction would not be constitutional. D For the insurance company, because suit must be brought in State B.

•B The court should rule for the purchaser's administrator on the minimum contacts issue and exercise in personam jurisdiction over the insurance company. Generally, fewer contacts will suffice for in personam jurisdiction if those contacts are directly related to the cause of action. Here, even though the sale of the insurance policy constituted only a single contact, it is the contact at issue in the case. Thus, the exercise of jurisdiction would be constitutional. (A) is incorrect because the exercise of jurisdiction, in addition to being authorized by statute, must also be constitutional. (C) is incorrect because a single contact is sufficient if the case is based on that contact, as discussed above. (This is called "specific jurisdiction.") (D) is incorrect, because a suit in State A would be permissible if it is authorized by statute and constitutional, which it is here.

What are the five ways minimum contracts are established?

(1) Entering the state/doing business (2) Entering into a contract with a resident of the forum state (3) By placing a product in the stream of commerce that meets either the stream of commerce or stream of commerce tests (4) By causing effects in the forum state, or (5) Property ownership.

28 U.S.C. 1367 (b) applies only to

Cases involving diversity. NOT federal questions.

Do minimum contacts have to be continuous and systematic?

Yes.

A used car dealer in State A is profiled in an article in a newspaper with national circulation. The article states that the dealer never sells a car without rolling the odometer back at least 25,000 miles. In fact, the dealer is an honest businessperson who never changes odometers, so he sued the newspaper for libel in State A state court. The newspaper's entire operation is conducted from its offices in State B, but it sells 5,000 copies in State A on an average day. In its initial pleading, the newspaper argues for dismissal based on lack of jurisdiction over the newspaper because of insufficient contacts. What is the likely ruling of the court on this issue? A the motion because the newspaper's contacts with State A were sufficient so it should reasonably anticipate being haled into court in State A. B Deny the motion, unless State A lacks a "doing business" jurisdictional provision. C Grant the motion, because selling 5,000 copies of a newspaper per day is not significant business. D Grant the motion, unless State A has a long arm statute covering the newspaper's conduct.

•A The court is likely to rule that the newspaper is subject to personal jurisdiction for a libel action in State A because it has sufficient minimum contacts with the state. The publisher should reasonably anticipate causing injury wherever the newspaper is sold, and so should reasonably anticipate being haled into court in each state. [Keeton v. Hustler Magazine, 465 U.S. 770 (1984); Calder v. Jones, 465 U.S. 783 (1984)] (B) and (D) are incorrect. The exercise of jurisdiction over a particular defendant must be both authorized by statute and constitutional. These are separate requirements. The newspaper's claim of insufficient contacts goes to the constitutionality of the exercise of jurisdiction. Whether State A has a "doing business" statute or a long arm statute is irrelevant to this determination. (C) is factually incorrect; selling 5,000 newspapers per day would probably be considered sufficient contact, especially since the cause of action arose from that conduct.

Ganja, a Pennsylvania citizen, filesalawsuit for $100,000 in the appropriate federal court against Matthews, Bernstein, Rollins and Grey, a 120-person law firm with offices in New York City. Most of the partners live in New York but ten live in New Jersey and one in Pennsylvania. The partners file a motion to dismiss. The court will A. Grant the motion since the there is no complete diversity. B.Grant the motion because most of the partners live in New York. C. Deny themotion because Ganja is a citizen of Pennsylvania, Matthews, Bernstein, Rollins and Grey is acitizen of New York and the amount in controversy is met. D. Deny the motion because Ganja can establish to a legal certainty that he suffered $100,000worthofdamages.

A

Imagine that during a driving trip from Pennsylvania to Maine, Donald takes a wrong turn and, unbeknownst to him, drives into Vermont. Before Donald discovers his error, he hits Penelope. Penelope sues Donald in Vermont, and Donald moves to dismiss the case for lack of personal jurisdiction. Assuming that Vermont's long arm statute authorizes jurisdiction over the action, the motion should be A. denied, because the case arises out of Donald's contacts in Vermont. B. granted. Personal jurisdiction would be unreasonable in this case, even if Donald's accident is considered to be a contact with Vermont. C. granted. Donald did not purposefully avail himself of the privilege of driving in Vermont because he drove into Vermont by mistake. As a result, he lacked any constitutionally cognizable contacts in Vermont. D. granted. Not only did Donald lack any contacts in Vermont, but the exercise of personal jurisdiction would be unreasonable under the circumstances.

A

Jeff Gordon from California sues Richard Petty from Florida for injuries sustained in an auto accident on I-4. Gordon Seeks $200, 000.Petty then brings a claim (under FRCP 14) against Joe Nemechek-his passenger-also from Florida, for the entire amount claimed by Gordon against him. Petty alleges that Nemechek grabbed the steering wheel of the vehicle and turned it to the right while stating that he saw something in the roadway. Gordon then filesa claim against Nemechek for $50,000 from the same accident for damages to his car. Does the court have subject matter jurisdiction over the entire lawsuit? A. No, although the court has diversity jurisdiction over Gordons' claim against Petty, it does not have supplemental jurisdiction over Gordon's claim against Nemechek. B.No, because both Petty and Nemecheck are from Florida. C. Yes, because the court has diversity jurisdiction over Gordon's claim against petty and supplemental jurisdiction overGordon's claim against Nemechek because it arises from the same common nucleaus of operative facts. D. Yes because defendants (Petty and Nemechek) are from Florida and Plaintiff (Gordon) is from California, therefore there is complete diversity of citizenship among the parties.

A

Karen has always lived in Kansas. She wants to try new experiences, and so she has decided to move to New York City. She has started getting home delivery of The New York Times and is looking for a job and an apartment. Karen loads up her Audi and begins the move to New York. While on the road, she is involved in a serious automobile accident and is hospitalized in Ohio. Where is Karen domiciled at this point? A.Kansas. This is her domicile of origin. B. Ohio. Given the extent of her injuries, she will be there indefinitely. C.New York. She has made the decision to make New York her home. D. Karen is in transit —at the moment she has no domicile.

A

Matthew McConaughey (alright, alright, alright!!) is a citizen of Texas. At age 18, he leaves Texas to attend college in Michigan. He knows he does not want to return to Texas, but is not sure where he wants to settle. After two years of enduring bone-chilling, teeth-chattering, finger-biting coldin Michiganduring winter, he tranfers to college in Florida. After two more years in sunny Florida, he graduates and attends medical college in Massachusetts. After four years there, he graduates and moves to Hawaii for a one year internship.That's where he currently is.During hismoves, he has never formed an intent to make these places his domicile, but he is certain that he does not want to go back to Texas, even though Texas is as it is in heaven. McConaughey's citizenship for diversity purposes is likely A. Texas. B.Michigan. C. Hawaii. D. Massachusetts.

A

Rose McGowan, a resident of California suffered emotional distress when defendant Jeffrey Epstein forcibly tried to kiss her, at the set of the TV series Charmed, in Hollywood California. Epstein resides in New York, owns a palatial mansion in West Palm Beach Florida and a private Caribbean Island in the Virgin Islands. When Epstein, while in California,heard that McGowan was going to sue him,he jumped up one day and cried out "I have formed the intent to make this state my permanent home!!!. McGowan has now sued Epsteinin the Federal District Court in LA. Assuming the amount in controversy is met, does the court have subject matter jurisdiction? A. No, becauseEpstein is domiciled in California. B. No, because Epstein is domiciled in New York. C. Yes, because Epstein is domiciled in New York, Florida and Virgin Islands. D. Yes, because there is complete diversity and the amount in controversy is met.

A

Susan Gordon, after growing up in Pennsylvania, goes to Ricks College, in Idaho, planning to get a two-year nursing degree and return to practice nursing in Pennsylvania. Two months after starting school, she files suit in a Pennsylvania federal district court against Dr. Rodriguez, a Pennsylvania citizen who treated her in Pennsylvania for an injury. She claims jurisdiction based on diversity. The court probably A. lacks diversity jurisdiction, because Gordon is still domiciled in Pennsylvania. B. lacks diversity jurisdiction, because the treatment took place in Pennsylvania. C. has diversity jurisdiction, because she was living in Idaho when she filed the suit. D. would have diversity jurisdiction, if she brought the action in an Idaho federal court.

A

You represent Floyd Mayweatherwho was injured in a bar fight. You would like to file the case in federal court in Massachusetts based on diversity jurisdiction. Assume that Mayweather is a citizen of Massachusetts based on his domicile there. Barristers is incorporated in Delaware. Itowns and operates eight bars in Massachusetts. It also owns and operates one in Rhode Island(where it also has its corporate headquarters), two in Connecticut, and one in Maine. A federal court would A. Have diversity jurisdiction, even if Barristers'principal place of business is in Massachusetts, because Mayweather is a citizen of Massachusettsand Barristers is a citizen of Delaware. B. Have diversity jurisdiction, because Mayweather is a citizen of Massachusetts and Barristersis a citizen of Rhode Island and Delaware. C. Lack diversity jurisdiction, because both Mayweather and Barristers are citizens of Massachusetts. D. Lack diversity jurisdiction, if the suit is brought in Massachusetts, since Mayweather is domiciled there.

A

Zoolander, from California and Mugatu, from Texas are injured in the same plane accident. They both sue Hansel from New York who owns the plane. Zoolander seeks $500,000 in damages to his face and hair, and Mugatu seeks $10, 000 in damages to his clothes. Would a federal court have subject matter jurisdiction over the entire case? A. Yes, because Zoolander meets the jurisdictional requirements, the court would have supplemental jurisdiction over Mugatu. B. Yes, because the claims are aggregated. C. No, because aggregation is not allowed. D. No, because 28 USC §1367(b) bars supplemental jurisdiction over Mugatu.

A

•A used car dealer in State A is profiled in an article in a newspaper with national circulation. The article states that the dealer never sells a car without rolling the odometer back at least 25,000 miles. In fact, the dealer is an honest businessperson who never changes odometers, so he sued the newspaper for libel in State A state court. The newspaper's entire operation is conducted from its offices in State B, but it sells 5,000 copies in State A on an average day. In its initial pleading, the newspaper argues for dismissal based on lack of jurisdiction over the newspaper because of insufficient contacts. •What is the likely ruling of the court on this issue? •A the motion because the newspaper's contacts with State A were sufficient so it should reasonably anticipate being haled into court in State A. •B Deny the motion, unless State A lacks a "doing business" jurisdictional provision. •C Grant the motion, because selling 5,000 copies of a newspaper per day is not significant business. •D Grant the motion, unless State A has a long arm statute covering the newspaper's conduct.

A

•A wholesaler sued a retailer in a federal court in State A. The retailer timely filed and served a motion to dismiss for lack of subject matter jurisdiction. The court denied this motion. •Thereafter, the retailer filed and served his answer. Ten days after serving his answer, the retailer filed an amended answer, raising, for the first time, the defense of lack of personal jurisdiction, which was available when the motion mentioned above was filed. •Should the court consider the personal jurisdiction objection? •A No, because that defense has been waived. •B No, because objections to personal jurisdiction may only be made by making a motion to dismiss before filing an answer. •C Yes, because the retailer may serve an amended answer as of right within 21 days after serving his original answer. •D Yes, because the amendment relates back to the original answer, thus preserving his right to raise the objection.

A

Specific jurisdiction gives the court the power to adjudicate: (A) causes of action arising out of or related to the defendant's contacts with the forum state. (B) all disputes between the plaintiff and the defendant, even if they are not related to the defendant's contacts with the forum state. (C) all disputes described with sufficient particularity in the plaintiff's complaint. (D) any cause of action between the plaintiff and the defendant, as long as one of the plaintiff's claims arises out of or relates to the defendant's contacts with the forum state.

A In specific jurisdiction cases, there must be some kind of relationship between the defendant's contacts with the forum and the plaintiff's cause of action. Courts often describe this by saying that the cause of action "arises out of" the contacts.

California's long arm statute subjected "foreign corporations to suit in California on insurance contracts with residents of that State even though such corporations [could not] be served with process within [California's] borders." Given that the statute clearly conferred personal jurisdiction in this case, why did the Court need to discuss the Constitution?

A court can exercise personal jurisdiction only if the relevant long arm statute authorizes it and the Constitution does not prohibit it. (Review the diagram at the end of the previous chapter.) Here, the California statute authorized personal jurisdiction, but the Court still had to determine whether the statute, as applied to the facts of this case, was constitutional.

What is Specific Jurisdiction?

An isolated act that exposes a company to a P's suit on the act. (Ex: Insurance Co sued by its only policyholder in a state. Jurisdiction is upheld b/c the forum has an interest in protecting citizens from out of state parties.) If a suit is not based on D's systematic or continuous contacts, then it must be based on specific jurisdiction. The defendant's contacts with the forum states are more limited. However, the claim involved must arise out of those contacts.

Pietra has a claim against Donnor, a California citizen, that arose out of a car accident that occurred in California. Pietra is a citizen of North Carolina and asks a North Carolina court to attach a money market account that Donnor owns in the state and that he opened while visiting the state. Pietra then seeks to litigate the car accident case in North Carolina, resigning herself to recover only the funds in Donnor's North Carolina money market account. Assume Donnor's North Carolina account is completely unrelated to Pietra's underlying claim against Donnor, and assume that Donnor does not otherwise have contacts with North Carolina sufficient to subject him to in personam jurisdiction in the state. Under these facts, most courts would probably A. not exercise quasi in rem jurisdiction because Donnor lacks sufficient contacts with the state. B. not exercise quasi in rem jurisdiction because this basis for jurisdiction was eliminated by the Supreme Court's Shaffer opinion. C. exercise quasi in rem jurisdiction as long as the state court attached Donnor's account at an appropriate time in the case. D. exercise in personam jurisdiction because the Supreme Court has eliminated the distinction between quasi in rem and in personam jurisdiction.

A is the best answer: There is no specific jurisdiction, so there is likely no quasi in rem jurisdiction.

What is minimum jurisdiction?

A term used to determine when it is appropriate for a court in one state to assert personal jurisdiction over a defendant. The requirement of minimum contacts means that D has to have taken actions that were purposefully directed towards the forum state.

The Mottleys may not have realized that their case, which raised several novel questions of federal law, might fail to satisfy the statutory requirement for federal question jurisdiction. Suppose that after the Supreme Court's decision the federal trial judge gave them the opportunity to amend their complaint to solve the problem. What should they plead to get into federal court? A. They could not amend the complaint in any way that would satisfy the statutory requirements for federal question jurisdiction. B. They should amend, stating with more specificity why they know the railroad would raise the federal statute as a defense. C. They should amend to allege that the railroad has deprived them of their constitutional rights under the Fifth Amendment. D. They should amend to allege a right to relief under the federal transportation statute itself, not just under the contract.

A. B is not going to do the trick. The Court says anticipating a defense by the railroad is not sufficient to make a case arise under federal law. So stating the expected defense more clearly, or alleging that you are really sure they will raise it, isn't going to change the fact that you are just anticipating a defense. C is a loser too. The railroad hasn't deprived the Mottleys of any constitutional rights; if anyone has it is Congress. The Fifth Amendment grants protection from governmental deprivations without due process, not railroad deprivations. Only their contract protects them from that. And D wouldn't work unless the federal statute had created a right to train passes that the Mottleys could enforce. But the statute didn't do that; it barred issuing the passes. So, it's hard to see how fancy pleading could help the Mottleys. Contract cases just don't arise under federal law (in the Mottley sense of that phrase). A is the best answer. The Mottleys could not manufacture federal question jurisdiction by clever pleading. Their only source of a right to relief against the railroad was the contract, so their counsel—even after reading the Supreme Court's Mottley decision—would not be able to create federal jurisdiction by redrafting their complaint.

Ganja, a Pennsylvania citizen, files a lawsuit for $100,000 in the appropriate federal court against Matthews, Bernstein, Rollins and Grey, a 120-person law firm with offices in New York City. Most of the partners live in New York but ten live in New Jersey and one in Pennsylvania. The partners file a motion to dismiss. The court will A. Grant the motion since the there is no complete diversity. B.Grant the motion because most of the partners live in New York. C. Deny the motion because Ganja is a citizen of Pennsylvania, Matthews, Bernstein, Rollins and Grey is a citizen of New York and the amount in controversy is met. D. Deny the motion because Ganja can establish to a legal certainty that he suffered $100,000 worth of damages.

A. Grant the motion since there is no complete diversity

Jeff Gordon from California sues Richard Petty from Florida for injuries sustained in an auto accident on I-4. Gordon Seeks $200, 000.Petty then brings a claim (under FRCP 14) against Joe Nemechek- his passenger- also from Florida, for the entire amount claimed by Gordon against him. Petty alleges that Nemechek grabbed the steering wheel of the vehicle and turned it to the right while stating that he saw something in the roadway. Gordon then files a claim against Nemechek for $50,000 from the same accident for damages to his car. Does the court have subject matter jurisdiction over the entire lawsuit? A. No, although the court has diversity jurisdiction over Gordons' claim against Petty, it does not have supplemental jurisdiction over Gordon's claim against Nemechek. B.No, because both Petty and Nemecheck are from Florida. C. Yes, because the court has diversity jurisdiction over Gordon's claim against petty and supplemental jurisdiction overGordon's claim against Nemechek because it arises from the same common nucleus of operative facts. D. Yes because defendants (Petty and Nemechek) are from Florida and Plaintiff (Gordon) is from California, therefore there is complete diversity of citizenship among the parties.

A. No, although the court has diversity jurisdiction over Gordon's claim against Petty, it does not have supplemental jurisdiction over Gordon's claim against Nemechek.

Relationship between a website/internet product and personal jurisdiction.

Ask whether the website owner intentionally directed electronic activity into the state and whther that activity gave rise to the cause of the action in the case. If yes, then there is personal jurisdiction.

Assume that Mr. and Mrs. Mas were both domiciled in Louisiana while in school, but after finishing their degrees they move to France, planning to live there indefinitely. After arriving in France, they bring suit on a state law claim against Perry in federal court in Louisiana. A. The court lacks jurisdiction, because Mrs. Mas is not a citizen of a state or a foreign citizen. B. The court lacks jurisdiction, because Mrs. Mas remains a citizen of Louisiana. C. The court has jurisdiction, since neither Mr. Mas nor Mrs. Mas is domiciled in Louisiana. D. The court has jurisdiction, because Mr. and Mrs. Mas have become foreign citizens. Thus, the case is between a state citizen, Mr. Perry, and citizens of a foreign state.

A. Once she moves to France, Mrs. Mas loses her Louisiana citizenship. She remains a U.S. citizen, but she is not a citizen of any state since she is domiciled in France.

Matthew McConaughey (alright, alright, alright!!) is a citizen of Texas. At age 18, he leaves Texas to attend college in Michigan. He knows he does not want to return to Texas, but is not sure where he wants to settle. After two years of enduring bone-chilling, teeth-chattering, finger-biting cold in Michigan during winter, he tranfers to college in Florida. After two more years in sunny Florida, he graduates and attends medical college in Massachusetts. After four years there, he graduates and moves to Hawaii for a one year internship. That's where he currently is. During his moves, he has never formed an intent to make these places his domicile, but he is certain that he does not want to go back to Texas, even though Texas is as it is in heaven. McConaughey's citizenship for diversity purposes is likely A. Texas. B.Michigan. C. Hawaii. D. Massachusetts.

A. Texas

Emmanuel Macron, aFrench citizen moves to Orlando Florida, to study law. While there, he falls in love with Zooey Deschanel, a dashing brunette from Florida. They get married while in law school. After finishing law school, they both move to France planning to live there indefinitely. After arriving in France, they both file suit against Barry law school, in the U.S. District Court for the Middle District ofFlorida, alleging that at the end of their first year in Barry, they showed large reductions in positive affect, life satisfaction, and overall well-being, and large increases in negative affect, depression, and physical symptoms.Assume that they have a legal claim and that the amount in controversy is met. As to their lawsuit A.The court lacks jurisdiction, because Zooey is not a citizen of a state or a foreign citizen B.The court lacks jurisdiction, because Zooey remains a citizen of Florida. C.The court has jurisdiction , since neither Zooey nor Macron is domiciled in Florida. D. The court has jurisdiction, because Zooey and Macron have become foreign citizens. Thus the case is between a state corporation, Barry law, and citizens of a foreign state.

A. The court lacks jurisdiction, because Zooey is not a citizen of a state or a foreign citizen

What is an unenumerated long-arm statute

Assert jurisdiction over any action of defendant and cast the entire decision of the appropriateness of jurisdiction on the minimum contacts test. (Most states do this)

Would it be constitutional for a court to exercise in personam jurisdiction in the following cases? If so, what type of in personam jurisdiction? Specific or general? A. Demo (the Pennsylvania company from the earlier note) sends one of its products by mail to Paula, a California citizen. The product injures Paula in California, and she sues Demo in California. B. Same as A, except that Paula sues Demo in Pennsylvania. C. Same as A, except that after her injury, Paula moves to Arizona and sues Demo there. Assume that Demo's only contact with Arizona is that it recently sent a catalog to a potential customer in the state. D. Same as C, except that the lawsuit is brought by the Arizona citizen as a result of fraudulent claims in the catalog sent to her in the state.

A. There would be specific jurisdiction over Demo in California. Demo made a deliberate contact with California (the sending of the package to the state), and the claim arises out of that voluntary contact with the state. B. There would be general jurisdiction over Demo in Pennsylvania because Demo is at home there. C. There would not be in personam jurisdiction in this case. There is no general jurisdiction, because Demo does not appear to have sufficient contacts with Arizona to be considered at home there. Rather, from what we can tell, Demo merely sent one catalog to the state.There is also no specific jurisdiction. Although Demo does have a contact in Arizona (it sent the catalog there), the claim does not arise out of that contact. Rather, the claim arises out of Demo's contact with California. As a result, there is no personal jurisdiction in Arizona for this case. D. Even though Demo's contacts with Arizona are exactly the same as they were in C, there would be personal jurisdiction in this case. The key difference is that this claim arises out of Demo's contacts in Arizona, whereas in C, the claim did not.

Rosario sues Demerest in state court, claiming that Demerest fired him based on his age, a violation of the federal Age Discrimination in Employment Act. The state court should A. hear the suit. B. hear the case only if the parties are not diverse. C. dismiss the case, because it arises under federal law and must be filed in federal court. D. dismiss the case, unless Congress has expressly authorized bringing such federal claims in state court.

A. state courts usually may hear cases within the federal subject matter jurisdiction. They have "concurrent jurisdiction" over cases arising under federal law, unless Congress specifies that a particular type of federal claim must be brought in federal court.

The court in Bensusan refers to a hypothetical case in which an irate citizen of New Jersey fires a bazooka at New York, hitting an unsuspecting New Yorker. Imagine that the injured New Yorker sues the New Jersey defendant in New York state court. Assuming that the defendant moves to dismiss the case on personal jurisdiction grounds and that the New York long arm statute is the same as described in Bensusan, the court would probably A. deny the motion because personal jurisdiction would be constitutional in this case. B. grant the motion because personal jurisdiction would be unconstitutional in this case. C. grant the motion because the long arm statute does not authorize personal jurisdiction in this case. D. deny the motion because the long arm statute applies to these facts, and personal jurisdiction would be constitutional.

According to Bensusan, subsection (2) of the long arm statute requires the defendant's act to have occurred in New York. In the bazooka case, it didn't. Moreover, subsection (3) does not apply here, because there is no evidence that the defendant derived substantial revenue from New York. Thus, D is probably incorrect.

Assume that a company called Demo Corporation is incorporated in Pennsylvania and has its principal place of business and all of its employees and facilities in Pennsylvania. Also imagine that Patti and Demo entered into a contract in Pennsylvania and that, after Demo breached the contract, Patti moved to Massachusetts and sued Demo for the breach in her new state. Assume that Demo has had only one contact with Massachusetts: one of its drivers drove through the state to make a delivery in Maine. Can Patti sue Demo in Massachusetts for the contract breach?

Although Demo has a contact in Massachusetts (one of its drivers previously entered the state), that contact does not support jurisdiction over Patti's claim. The key to specific jurisdiction is that the plaintiff's claim must arise out of the particular contact that the defendant had in the state. Here, unlike in Hess, Demo's act of driving in Massachusetts was unrelated to the subject of the lawsuit. Patti's claim concerns an alleged breach of contract, not Demo's drive through Massachusetts. As a result, there is no specific jurisdiction over Demo in Massachusetts for Patti's lawsuit, even though the type of contact is exactly the same as it was in the Hess case.

Karen has always lived in Kansas. She wants to try new experiences, and so she has decided to move to New York City. She has started getting home delivery of The New York Times and is looking for a job and an apartment. Karen loads up her Audi and begins the move to New York. While on the road, she is involved in a serious automobile accident and is hospitalized in Ohio. Where is Karen domiciled at this point? ​ ​ (A) Kansas. This is her domicile of origin. ​ ​ (B) Ohio. Given the extent of her injuries, she will be there indefinitely. ​ ​ (C) New York. She has made the decision to make New York her home. ​ ​ (D) Karen is in transit — at the moment she has no domicile.​

Answer (A) is correct. The statement that Karen has always lived in Kansas indicates that Kansas is her domicile of origin. She has the required intent to make New York her home. However, she lacks the second requirement for acquiring a domicile of choice: presence at the new domicile. A person keeps her old domicile until she acquires a new one, and because she did not yet reach New York, she is still domiciled in Kansas.​

Greg, from Georgia, wants to sue Frank, from Florida, for personal injuries totaling $250,000. Must he file the suit in federal court? ​ ​ (A) Yes. The complete diversity requirement has been met. ​ ​ (B) Yes. This is necessary to protect Frank from local prejudice. ​ ​ (C) No. While federal jurisdiction would be proper, it is not exclusive. ​ ​ (D) No. Only federal question jurisdiction is exclusive in the federal courts.​

Answer (C) is correct. This claim could properly be brought in federal court based on diversity jurisdiction, because the parties are from different states and the amount in controversy is sufficient. Diversity jurisdiction, however, is just an option for the parties. The federal courts have concurrent jurisdiction with the state courts in diversity cases, and so these lawsuits can also be filed in state court. (Under certain circumstances, defendant sued in state court may choose to "remove" the case to federal court. ​

What is a judgment as a matter of law?

Arguing that the opponent's evidence on some element of the claim or defense is too weak to allow a verdict in the opponent's favor

ABC Corporation ("ABC") based in Florida sues Ed, its former employee, also a resident of Florida, for business libel, a tort claim. The suit is brought in federal district court and alleges that Ed falsely reported to a federal agency that ABC was using substandard materials on federal construction project.Ed answers admitting that he made a report to the agency but alleging that his report was protected by the First Amendment. He also files a counter claim against ABC based on the Federal Whistle Blowers Act, which authorizes damages for anyone who is dismissed or disciplined for reporting fraud on the federal government. Ed files a motion to dismiss for lack of subject matter jurisdiction. The court is likely to A.Grant Ed's motion because both parties are residents of Florida. B. Grant Ed's motion because under the well pleaded complaint ruleand Holmes creation test, ABC's filing does not arise under federal law. C. Deny Ed's motion because under the well pleaded complaint rule and Holmes creation test, ABC's filing does not arise under federal law. D. Deny Ed's motion because he counter claimed against ABC based on the Federal Whistle Blowers Act, which authorizes damages for anyone who is dismissed or disciplined for reporting fraud on the federal government.

B

Allen Iversonis from Hampton Virginia. Hisluggage containing jewelry and other property allegedly worth over $50,000 was taken from a baggage room at a Las Vegas, Nevada hotel at which he was a guest. Iverson sued the hotel, in an appropriate federal court,for negligence, breach of bailment, and other violations of its duties as innkeeper. Iverson values the missing property as worth over $80,000. After completion of discovery, the Hotel filed a motion to dismiss based upon Nev. Rev. Stat. § 651.010 contending that the statute limits the common-law liability of innkeepers (owners of hotels) for the theft, loss, damage, or destruction of any guest's property to $750. The motion will likely be A. Granted because Iverson's luggage containing jewelry and other property was allegedly worth over $50,000 but less than $75,000, the required amount in controversy. B. Granted because the court lacks subject matter jurisdiction. C. Denied because under the common law, an innkeeper is responsible for the safety of property entrusted to his care by a guest. D. Denied because Iverson values the missing property as worth over $80,000.

B

Dr. Doofenshmirtz (Defendant) calls Perry the Platypus (Plaintiff) a d bag in front of Phineas and Ferb. Plaintiff files an action for defamation and intentional infliction of emotional distress against Defendant in state court. Defendant timely files notice of removal to federal court contending that the First Amendment protects his right to free speech. What result? A. Defendant will remove the case because his cause of action is grounded in federal law. B. Defendant will not remove the case since the existence of a federal defense is insufficient to confer original federal jurisdiction. C. Defendant will remove the case because state courts and federal courts have concurrent jurisdiction. D. Defendant will not be able to remove because federal courts are courts of limited jurisdiction.

B

Honey Boo Boo grew up in Erie Pennsylvania. In August 2017, she departed for Law school in Orlando FL. After starting law school in FL, she registered her car there and established a bank account there. She gets health coverage through the college. She has a crib there, which she has taken on a one year lease. She has a part time job in Orlando Boys and Girls Clubteaching kids how to translate"hillbilly English"into "BougieEnglish". She has FLtaxes(assume Florida has income tax) taken out of her pay check. On her application for aclerkship with the Orange countycourt, she listed her Orlandoaddress and telephone number. Her plan is to complete law school and return to Pennsylvaniawhere her fiancée lives. They have reserved a hall for a wedding after she finishes law school. However, she may need more than three years to finish law schoolsince her interest in reality TV is taking a lotof her time. Three months after arriving in Orlando, HoneyBoo Boo brings a medical malpractice suit against a Pennsylvania cosmetic surgeon for a face-lift gone bad. She sues in federal courtin Orlandoonthe basis of diversity. The court will probably conclude that Honey Boo Boo is domiciled in ? A. Pennsylvania, since she is only in Floridaas a student. B. Pennsylvania,since she intends to return to Pennsylvania when she finishes her degree. C. Florida, since she has an apartment, goes to school, has a job andhealth coverage there, and pays Florida stateincome taxes. D. Florida, since she may need more than three years to finish law school.

B

•A citizen of State A purchased life insurance by mail from a State B insurance company. The policy was the only one that the company had ever sold in State A. The purchaser mailed premiums from State A to State B for five years, and then died. The insurance company refused to pay the policy benefits. The purchaser's administrator sued the company in State A state court. The state has a long arm statute that grants a state court in personam jurisdiction over a defendant who "contract[s] to insure any person, property, or risk located within this State at the time of the contracting." The insurance company argued that its only contact with State A since it began its business was the purchaser's insurance policy, and that this single contact does not meet the minimum required for the exercise of in personam jurisdiction under International Shoe. •How should the court rule on the minimum contacts issue? •A For the purchaser's administrator, because the State A statute authorizes jurisdiction. •B For the purchaser's administrator, because of the close connection between the contact and the case. •C For the insurance company, because the exercise of jurisdiction would not be constitutional. •D For the insurance company, because suit must be brought in State B

B

Massive Mining Corporation (MMC) was located in West Virginia and did all of its business there. Unfortunately, after a major mine collapse, MMC had to suspend its mining operations. Due to local hostility, the company's management decided to move the company headquarters to Vermont. The CFO opened a bank account in Vermont and paid all the bills there. Dan Blankenboat, the CEO, bought a house in the mountains of Vermont, and any board meetings and management team conferences that happened after the collapse occurred there. It is unknown at this time when MMC will be able to re-open the mine and move back to West Virginia. Morgan, a miner who worked for MMC, was killed when the mine collapsed. His widow has moved to New Hampshire, so the possibility of suing MMC in Vermont is very appealing. Could Vermont constitutionally assert jurisdiction over MMC for a claim arising out of the collapse of the mine? (A) Yes. MMC has a level of contacts with Vermont that would satisfy the Supreme Court's test for specific personal jurisdiction. (B) Yes. MMC now has continuous and systematic contacts with Vermont. (C) No. MMC's contacts with Vermont do not involve mining. (D) No. Morgan's injuries did not arise out of MMC's contacts with Vermont.

B The facts of this problem are similar to those of Perkins v. Benguet Consol. Mining Co., 343 U.S. 917 (1952), the only case in which the Supreme Court has upheld jurisdiction based on contacts unrelated to the forum. The Court limited jurisdiction to situations in which the defendant corporation's affiliations with the forum state are "so continuous and systematic" as to render it "essentially at home in the forum state," i.e. comparable to a domestic enterprise in that state

Goggles.com is a successful e-commerce business. It sold a computer program to Winkle; sadly, the program did not quite meet the expectations created by the website, and Winkle wants his money back. Winkle had been at home in Massachusetts when he discovered Goggles.com's business while surfing the net, he was in Massachusetts when he downloaded the program, and he used his credit card with a Massachusetts billing address to pay for the program. Assume that Goggles.com's website contained no choice of forum clause, and that other than whatever sales it might make from its website (and it has made many to Massachusetts residents), Goggles.com has no contacts with Massachusetts. Can a court in Massachusetts constitutionally assert jurisdiction over Winkle's claim against Goggles.com? (A) Even if Goggles.com's website had been purely passive (just pictures of its products), that website makes it present in Massachusetts and Winkle can sue it there. (B) Because Goggles.com's website is extremely interactive, allowing customers to learn about merchandise and to purchase it online, Massachusetts can assert jurisdiction over Goggles.com. (C) Because Goggles.com's website is constantly available in the state of Massachusetts, it constitutes continuous and systematic contact and justifies general jurisdiction over Goggles.com, even if it never makes a sale to a resident of Massachusetts. (D) No, internet contacts take place only in virtual space and cannot provide the basis for contact-based jurisdiction.

B Zippo Co. v. Zippo Dot Com, 952 F. Supp. 1119 (W.D. Pa. 1997), is often cited as the source of this test. A purely passive web page (just pictures and text, with little or no interactivity and no way to purchase a product online) will not provide the contacts needed for personal jurisdiction. The customer generally will be regarded as having come to the website, rather than the website being regarded as having sought to sell to the forum residents. At the other end of the scale, a commercial website through which the merchant conducts business over the Internet with forum residents will often support jurisdiction. Goggles.com may fall into this category, particularly if it has a high volume of sales to forum residents.

Pietra has a claim against Donnor, a California citizen, that arose out of an injury that Pietra suffered while jogging on a rental property in North Carolina that Donnor owned. Pietra sues Donnor in North Carolina and asks the North Carolina court to attach Donnor's property at the outset of the case in an attempt to establish quasi in rem jurisdiction. Donnor's property is valuable enough to cover Pietra's damages if she wins. Assume that Donnor purchased the property only to rent it out and has never been to North Carolina. (Donnor purchased the property from California.) Donnor has no other contacts with North Carolina. Donnor moves to dismiss the case for lack of personal jurisdiction. Under these facts, most courts would probably A. grant the motion, because Donnor has never been to North Carolina. B. deny the motion, because this case arises out of Donnor's contacts with North Carolina. C. deny the motion, because Donnor is at home in North Carolina and is thus subject to general jurisdiction there. D. grant the motion, because this case does not arise out of Donnor's contacts with North Carolina.

B, therefore, is the best answer. The Court said in Shaffer: "The presence of property may also favor jurisdiction in cases such as suits for injury suffered on the land of an absentee owner, where the defendant's ownership of the property is conceded but the cause of action is otherwise related to rights and duties growing out of that ownership."

Snoop Dogg (a california citizen) sues Ford Motor Co. (a Delaware corporation with its principal place of business in Detroit, Michigan) in a state court in Orlando Florida for products liability for $200,000. Snoop was injured when his Ford Bronco fell apart in Las Vegas Nevada. Ford may remove to which district(s)? A.The United States District Court for the District of Nevada B. The United States District Court for the Central District of California C. The United States District Court for the District of Delaware OR The United States District Court for the Eastern District of Michigan D. The United States District Court for the Middle District of Florida

D

ABC Corporation ("ABC") based in Florida sues Ed, its former employee, also a resident of Florida, for business libel, a tort claim. The suit is brought in federal district court and alleges that Ed falsely reported to a federal agency that ABC was using substandard materials on federal construction project.Ed answers admitting that he made a report to the agency but alleging that his report was protected by the First Amendment. He also files a counter claim against ABC based on the Federal Whistle Blowers Act, which authorizes damages for anyone who is dismissed or disciplined for reporting fraud on the federal government. Ed files a motion to dismiss for lack of subject matter jurisdiction. The court is likely to A.Grant Ed's motion because both parties are residents of Florida. B. Grant Ed's motion because under the well pleaded complaint rule and Holmes creation test, ABC's filing does not arise under federal law. C. Deny Ed's motion because under the well pleaded complaint rule and Holmes creation test, ABC's filing does not arise under federal law. D. Deny Ed's motion because he counter claimed against ABC based on the Federal Whistle Blowers Act, which authorizes damages for anyone who is dismissed or disciplined for reporting fraud on the federal government.

B. Grant Ed's motion because under the well pleaded complaint rule and Holmes creation test, ABC's filing does not arise under Federal Law

Allen Iverson is from Hampton Virginia. His luggage containing jewelry and other property allegedly worth over $50,000 was taken from a baggage room at a Las Vegas, Nevada hotel at which he was a guest. Iverson sued the hotel, in an appropriate federal court, for negligence, breach of bailment, and other violations of its duties as innkeeper. Iverson values the missing property as worth over $80,000. After completion of discovery, the Hotel filed a motion to dismiss based upon Nev. Rev. Stat. § 651.010 contending that the statute limits the common-law liability of innkeepers (owners of hotels) for the theft, loss, damage, or destruction of any guest's property to $750. The motion will likely be A. Granted because Iverson's luggage containing jewelry and other property was allegedly worth over $50,000 but less than $75,000, the required amount in controversy. B. Granted because the court lacks subject matter jurisdiction. C. Denied because under the common law, an innkeeper is responsible for the safety of property entrusted to his care by a guest. D. Denied because Iverson values the missing property as worth over $80,000.

B. Granted because the court lacks subject matter jurisdiction

What is transient jurisdiction?

Refers to personal jurisdiction over a defendant who is served with process while in the forum state only temporarily such as during travel."Transient or "tag" Jurisdiction. CB 286 "Transient Jurisdiction refers to personal jurisdiction over a defendant who is served with process while in the forum state only temporarily such as during travel.

You represent Corey in his bar fight case and would like to file the case in federal court based on diversity jurisdiction. Assume that Corey is a citizen of Massachusetts, based on his domicile there. Barristers is incorporated in Delaware. It owns and operates eight bars in Massachusetts. It also owns and operates one in Rhode Island (where it also has its corporate headquarters), two in Connecticut, and one in Maine.A federal court would A. have diversity jurisdiction, even if Barrister's principal place of business is in Massachusetts, because Corey is a citizen of Massachusetts and Barristers is a citizen of Delaware. B. have diversity jurisdiction, because Corey is a citizen of Massachusetts and Barristers is a citizen of Rhode Island and Delaware. C. lack diversity jurisdiction, because both Corey and Barristers are citizens of Massachusetts. D. lack diversity jurisdiction, if the suit is brought in Massachusetts, since Corey is domiciled there.

B. PPB=Headquarters=Rhode Island

Honey Boo Boo grew up in Erie Pennsylvania. In August 2017, she departed for Law school in Orlando FL. After starting law school in FL, she registered her car there and established a bank account there. She gets health coverage through the college. She has a crib there, which she has taken on a one year lease. She has a part time job in Orlando Boys and Girls Club teaching kids how to translate"hillbilly English"into "BougieEnglish". She has FL taxes(assume Florida has income tax) taken out of her pay check. On her application for a clerkship with the Orange county court, she listed her Orlando address and telephone number. Her plan is to complete law school and return to Pennsylvania where her fiancée lives. They have reserved a hall for a wedding after she finishes law school. However, she may need more than three years to finish law school since her interest in reality TV is taking a lot of her time. Three months after arriving in Orlando, HoneyBoo Boo brings a medical malpractice suit against a Pennsylvania cosmetic surgeon for a face-lift gone bad. She sues in federal court in Orlando on the basis of diversity. The court will probably conclude that Honey Boo Boo is domiciled in ? A. Pennsylvania, since she is only in Florida as a student. B. Pennsylvania, since she intends to return to Pennsylvania when she finishes her degree. C. Florida, since she has an apartment, goes to school, has a job and health coverage there, and pays Florida state income taxes. D. Florida, since she may need more than three years to finish law school.

B. Pennsylvania, since she intends to return to Pennsylvania when she finishes her degree

How do you waive consent to personal jurisdiction?

By waiving objection on the basis of lack of personal jurisdiction or by failing to object on that ground in a timely fashion. A defendant normally has 21 days after being served with the summons and complaint to file an answer or to object to jurisdiction. An objection not waived is raised.

Cannavo, from Oregon, sues Singh, from Minnesota, after he is fired by Singh three months into a one-year contract as an office manager for Singh. He sues in an Oregon state court, seeking $200,000 for breach of contract. Three months later, Cannavo amends the complaint to add a claim under the Americans with Disabilities Act, a federal statute, claiming that Singh failed to make reasonable accommodations for a disability that interfered with his job performance. Two weeks after receiving the amended complaint, Singh removes the case to federal court. Which of the following is correct? A. Removal is proper under 28 U.S.C. § 1446(b)(3) after the amendment to add the federal claim. B. Removal is proper under the Mottley rule, because Cannavo seeks relief under federal law. C. Removal is not proper, because three months have gone by since the case was filed in state court. D. Removal is not proper, because the case could have been removed as originally filed.

Because Singh could have removed the original case based on diversity, but did not, he waived his right to remove. D is right. Ironically, Singh may have been content to litigate a contract claim in state court, but strongly prefer a federal forum once a federal claim is added to the case. However, he will not get one in this case.

Paula, a California citizen, sues David, an Illinois citizen, in federal district court in California. Paula alleges that while she was living in Illinois, David shot off fireworks that set Paula's house on fire. Assume that Paula's claim is based on a state law cause of action and that the alleged damages exceed $75,000. Also assume that David has never set foot in California and that California's long arm statute reaches as far as the Constitution allows. David moves to dismiss for lack of personal and subject matter jurisdiction. Which of the following statements is the most accurate? A. The court cannot hear the case because it lacks subject matter jurisdiction. B. The court cannot hear the case because it lacks personal jurisdiction. C. The court cannot hear the case because it lacks both personal and subject matter jurisdiction. D. The court can hear the case because it has subject matter jurisdiction and personal jurisdiction. E. The court can hear the case because it has subject matter jurisdiction, even though it lacks personal jurisdiction.

Because the court lacks personal jurisdiction but has subject matter jurisdiction, B is the answer.

Imagine that Ohio adds a new provision to its long arm statute, specifying that "a defendant is subject to personal jurisdiction in Ohio if the plaintiff resides in the state." Percy, an Ohio citizen, visited Georgia and was in a car accident with Donnor, a Georgia citizen. Donnor has never left Georgia and has no contacts in any other state. Percy returns home to Ohio and sues Donnor in Ohio state court. Donnor is properly served at his home in Georgia. Donnor immediately moves to dismiss the case on personal jurisdiction grounds (i.e., he does not waive the issue). The court should A. grant the motion because the exercise of personal jurisdiction is unconstitutional. B. deny the motion because the long arm statute authorizes personal jurisdiction, and personal jurisdiction in this case is constitutional. C. deny the motion because, although personal jurisdiction would not be constitutional on these facts, Ohio has authorized personal jurisdiction under its long arm statute. D. grant the motion because personal jurisdiction is not authorized by the long arm statute.

Because the exercise of personal jurisdiction would be unconstitutional here, the court must grant the motion. A, therefore, is the best answer.

After making tons of money in her reality TV gig, Honey Boo Boo decides to move from Georgia to LA California. She intends to permanently reside in LA. While driving through Arizona on her way to LA, her Mercedes c-class coupe, worth $95, 000 is hit by Mario Andretti's car. The Mercedes is totaled. Andretti is domiciled in California. Honey Boo Boo files a diversity suit in the Northern District of Arizona where the accident occurred. Mario Andretti files a motion to dismiss for want of personal jurisdiction. Judgment for whom? A- Honey Boo Boo because a substantial part of the events giving rise to the lawsuit happened in Arizona. B-Mario Andretti since Honey Boo Boo intends to permanently reside in LA, there is neither diversity jurisdiction nor minimum contacts. C- Honey Boo Boo because there is diversity jurisdiction and minimum contacts with the forum state. D.Mario Andretti because bringing the lawsuit in Arizona will offend the traditional notions of fair play and substantial justice.

C

After making tons of money in her reality TV gig, Honey Boo Boo decides to move from Georgia to LA California. She intends to permanently reside in LA. While driving through Arizona on her way to LA, her Mercedes c-class coupe, worth $95, 000 is hit by Mario Andretti's car. The Mercedes is totaled. Andretti is domiciled in California. Honey Boo Boo files a diversity suit in the Northern District of Arizona where the accident occurred. Mario Andretti files a motion to dismiss for want of personal jurisdiction. Judgment for whom? A. Honey Boo Boo because a substantial part of the events giving rise to the lawsuit happened in Arizona. B. Mario Andretti since Honey Boo Boo intends to permanently reside in LA, there is neither diversity jurisdiction nor minimum contacts. C. Honey Boo Boo because there is diversity jurisdiction and minimum contacts with the forum state. D.Mario Andretti because bringing the lawsuit in Arizona will offend the traditional notions of fair play and substantial justi

C

Ariana Grande, a citizen of Florida asserted a state law claim of $80,000 against Tyler Perry, a citizen of Georgia in federal court. Perry has a state law claim against Honey Boo Boo for $90,000 that arose out of the same transaction or occurrence as the original claim. As a result, Perry brought a third party claim against Honey Boo Boo.After learning of the third-party claim, Ariana Grande decided to bring her own state law claim against Honey Boo Boo for $90,000 and amended her original complaint accordingly. Does the federal court have subject matter jurisdiction over Ariana Grande's claim against Honey Boo Boo? A. No, because the amount in controversy is insufficient. B. No, because there is no diversity of citizenship. C. Yes, because the requirements of diversity have been met. D. Yes, because the court has supplemental jurisdiction.

C

Bob Marley, a citizen of New York, files a complaint against Legalize Ganja, also a citizen of New York, in federal court. The complaint consists of two counts: violation of Marley's patent rights and negligence. Marley's patent claim involves his allegation that Legalize Ganja marketed and sold "Bong Hits 4 Jesus" a product that Marley claims he holds a patent for. Marley's negligence claim involves the injuries Marley sustained when he slipped on an icy path on Legalize Ganja's property. Does the court have supplemental jurisdiction over the negligence claim? A. Yes, because the federal court has exclusive jurisdiction over the patent claim. B. Yes, because it is state law claim. C. No, because the two claims do not arise from the same underlying facts. D. No, because it is a state law claim.

C

Bode Miller, who lives in NH hit Lindsey Vonn, with a snowmobile while on vacation in Vonn's home state of MN, causing damages in excess of $80,000. Although Vonn has never been to NH, she sued Miller in NH federal district court. Which of the following statements is true regarding Vonn's NH lawsuit? A)The NH federal court lacks personal jurisdiction over the parties because Vonn has insufficient contacts with NH.(B)The NH federal court is an improper venue because the claim did not arise there. (C)The NH federal court may exercise personal jurisdiction over the parties because Vonn consented to such personal jurisdiction. (D) Vonn waived her claim against Miller by filing it in the wrong forum.

C

In Keeton v. Hustler Magazine, Inc., 465 U.S. 770 (1984), Keeton sued Hustler Magazine, a nationally circulated publication, for libel in New Hampshire. Although Keeton was a citizen of New York whose only connection to New Hampshire was that the libelous magazine had been distributed there, she sued Hustler in New Hampshire because it was the only state where the statute of limitations for her claim had not yet expired. Keeton's libel claim alleged damages to her reputation in New Hampshire and nationally, though her damages in New Hampshire were relatively small. What would you expect the Court to say about whether there was personal jurisdiction in this case? A. There isn't personal jurisdiction. Keeton's contacts with New Hampshire were insufficient to give rise to personal jurisdiction there. B. There isn't personal jurisdiction. Hustler's contacts with New Hampshire—and Keeton's damages there—were insufficient to give rise to jurisdiction. C. There is personal jurisdiction. The claim arises out of Hustler's contact with the state of New Hampshire.

C

Jerry Sandusky is served with a complaint by one of his sexual abuse victims. Sandusky answers by denying all allegations. 10 days later, he then moves to dismiss under 12(b)(2) for lack of jurisdiction over the person. Assuming that he is correct about jurisdiction, how should the court rule? A. Grant the motion because Sandusky is correct about jurisdiction. B. Deny the motion because Sandusky is correct about jurisdiction. C. Deny the motion under the 12 (h) (1) waiver rule. D. Grant the motion under the amended pleading rule.

C

Pluto files an action against Goofy in state court. Pluto faxes Goofy a courtesy copy of the complaint on October 1, 2013. On October 15, 2013, Pluto serves Goofy with the complaint and summons. On November 13, 2013 Goofy removes the case to Federal court. Pluto opposes contending that Goofy removed more than thirty days after receiving the faxed copy. What result? A.Pluto prevails because Goofy's removal is untimely. B.Pluto prevails because he timely faxed the complaint to Goofy. C. Goofy prevails because he removed within 30 days of being served with the complaint and summons. D. Goofy prevails because he removed the case within one year.

C

Snoop Dogg (a citizen of California) plans to sue Defendant Ford Motor Co., (incorporated in Delaware with its principal place of business in Michigan) for injuries suffered in an auto accident. Snoop believes that problems with the Ford F-350 series vehicle caused the accident. If Snoop sues Ford in California state court seeking$200,000 damages, can Ford remove the case to federal court? A. No, because the well-pleaded complaint rule indicates that a plaintiff is the master of his complaint. B. Yes, because defendants can always remove from state court to federal court. C. No, because no federal question is involved. D. Yes, because the federal court would have had original jurisdiction over Snoop's claim had Snoop filed there originally.

D

Stormy Daniels, a citizen of Louisiana has filed a breach of contract action against Trump Inc., in the federal district court in Louisiana seeking $130,000 in damages. Trump Inc., is a New York corporation whose principal place of business is New Orleans, Louisiana. Trump Inc., has filed a motion to dismiss for lack of subject matter jurisdiction. How should the federal court rule on the motion? (A). Deny the motion, because the amount in controversy is met. (B) Deny the motion, because the federal court has subject matter jurisdiction over the action based on its diversity of citizenship jurisdiction. (C) Grant the motion, because the federal court lacks subject matter jurisdiction. (D) Grant the motion because the federal court has discretion to either hear the action or dismiss it.

C

Suzzy from MA sues Danny from CT, in MA federal district court under diversity for $250,000 in injuries after a car accident. Danny counterclaims against Suzzy for $1,000 in damages to his car. Does Danny's counter claim fall within the court's supplement jurisdiction? A. No, because federal courts disregard potential defenses and counterclaims B. No, because it does not meet the amount in controversy requirement. C. Yes, because it is factually related to Suzzy's claim, even though Danny would not be able to file the $1,000 suit in federal court on its own D. Yes, because Danny's counterclaim arises under §1332

C

Tyler Perry lives and works in Georgia. He brought this suit against "The Gutter Press , Inc.", its local distributing company, and the reporter for libel, invasion of privacy, and intentional infliction of emotional distress. He filed the suit in the Northern District of Georgia. The Gutter Press , Inc is a Florida corporation with its principal place of business in Florida. It publishes a national weekly newspaper with a total circulation of over 5 million. About 600,000 of those copies, almost twice the level of the next highest State, are sold in Georgia. Tyler Perry's claims are based on an article that appeared in The Gutter Press Inc's September 8, 2017 issue. The article alleged that Tyler is a womanizer and fraudster, whose "Madea" movies are "a piece of shit". This characterization has harmed Tyler Perry' s reputation among his peers. The reporter contends that he did most of his research in Florida, relying on phone calls to sources in Georgia for the information contained in the article. He has only made one trip to Georgia. Shortly before publication, the reporter called Tyler Perry's home and read to him a draft of the article so as to elicit his comments upon it. Aside from his one trip and phone calls, the reporter has no other relevant contacts with Georgia.In a motion to dismiss by the reporter on the basis of lack of personal jurisdiction, the court is likely to find A.No jurisdiction because the reporter is not responsible for the circulation of the article in Georgia B.No jurisdiction because he did not purposefully avail himself of the benefits and protections of GA C.There is jurisdiction because he must reasonably anticipate being haled into court in Georgia" to answer for the truth of the statements made in their article. D. There is jurisdiction because venue is proper.

C

Tyler Perry lives and works in Georgia. He brought this suit against "The Gutter Press , Inc.", its local distributing company, and the reporter for libel, invasion of privacy, and intentional infliction of emotional distress. He filed the suit in the Northern District of Georgia. The Gutter Press , Inc is a Florida corporation with its principal place of business in Florida. It publishes a national weekly newspaper with a total circulation of over 5 million. About 600,000 of those copies, almost twice the level of the next highest State, are sold in Georgia. Tyler Perry's claims are based on an article that appeared in The Gutter Press Inc's September 8, 2017 issue. The article alleged that Tyler is a womanizer and fraudster, whose "Madea" movies are "a piece of shit". This characterization has harmed Tyler Perry' s reputation among his peers. The reporter contends that he did most of his research in Florida, relying on phone calls to sources in Georgia for the information contained in the article. He has only made one trip to Georgia. Shortly before publication, the reporter called Tyler Perry's home and read to him a draft of the article so as to elicit his comments upon it. Aside from his one trip and phone calls, the reporter has no other relevant contacts with Georgia.In a motion to dismiss by the reporter on the basis of lack of personal jurisdiction, the court is likely to find A.No jurisdiction because the reporter is not responsible for the circulation of the article in Georgia B.No jurisdiction because he did not purposefully avail himself of the benefits and protections of GA C.There is jurisdiction because he must reasonably anticipate being haled into court in Georgia" to answer for the truth of the statements made in their article. D. There is jurisdiction because venue is proper.

C

Really Big Eggs (RBE) is a multinational corporation that owns fast food outlets throughout the United States and the world. The company believes in concentrating its management structure in a single location, however, and so despite the far flung nature of its business all company officers work in the company's Illinois headquarters. The company is also incorporated in Illinois, and Illinois has more outlets per capita than any other state in the nation. Little, a resident of Nebraska, gets salmonella while on vacation in San Diego, and she thinks she got it from undercooked eggs at an RBE outlet. Little has heard that juries in Illinois are really sympathetic. Can Illinois courts constitutionally assert jurisdiction over RBE on Little's claim that they caused her food poisoning in California? (A) No. RBE's contacts with Illinois are not related to Little's cause of action. (B) No. There is no such thing as general jurisdiction over corporations. (C) Yes. Although there is some question about the extent of contacts that justifies a finding of general jurisdiction over a corporation, the contacts here are enough. (D) Yes. Because RBE is a huge corporation it can be sued anywhere.

C The defendant corporation is sued at the location of its principal place of business (comparable to suing a human in his home state), and the cause of action is similar to one which could have arisen from its Illinois business (all involve fast food sales).

Sydney Carton traveled to Paris, Texas, on vacation. While shopping at a local department store there (a "mom and pop" establishment that has never advertised outside of Texas), he was wrongfully detained and searched. Back home in Virginia, Carton filed suit against the store. Can a Virginia court constitutionally assert jurisdiction over Carton's claim against the Texas store? (A) Yes. The store intentionally injured a resident of Virginia. (B) Yes. Plaintiffs can always file suit in their home states. (C) No. The store's contacts with Virginia are insufficient to permit the assertion of jurisdiction. (D) Yes. The store profited from selling its products to a Virginian.

C The Texas store may have wronged Carton, but it did so at its home in Texas. It does not have sufficient contacts with Virginia to be sued there. Nor did the store take any purposeful act aimed at Virginia or seek to serve the Virginia market.

Snopp Dogg (a citizen of California) plans to sue Defendant Ford Motor Co., (incorporated in Delaware with its principal place of business in Michigan) for injuries suffered in an auto accident. Snoop believes that problems with the Ford F-350 series vehicle caused the accident. If Snoop sues Ford in California state court seeking $200,000 damages, can Ford remove the case to federal court? A. No, because the well-pleaded complaint rule indicates that a plaintiff is the master of his complaint B. Yes, because defendants can always remove from state court to federal court C. No, because no federal question was involved D. Yes, because the federal court would have had original jurisdiction over Snoop's claim had Snoop filed there originally.

D

Lacey Wildd, aka "human barbie", a citizen of West Virginia sues Ganja, a citizen of Jamaica, in federal district court in West Virginia after being knocked down by Ganja in Charleston airport. Ganja was running to the check in gate when he suddenly bumped into Lacey. Lacey alleges that the fall perforated her silicone breasts and broke her hips and she can no longer have sex. She is seeking $100,000 in damages. Convinced that Lacey is exaggerating her injuries, Ganja files a motion to dismiss for lack of subject matter jurisdiction. The federal court A. Will dismiss the case because there is no diversity. B. Will dismiss the case because based on the facts, it is legally certain that Lacey will not recover the jurisdictional amount. C. Will hear the case because it is apparent on the face of the complaint that Lacey can recover the jurisdictional amount and the parties are diverse. D. Will hear the case because there is diversity.

C- Take the P's allegations as true. It is up to the D to prove that someone cant reach the AIC, not the court. Only ask whether the P can recover $75K, not the amount they are asking for if it is over $75K

Bode Miller, who lives in NH hit Lindsey Vonn, with a snowmobile while on vacation in Vonn's home state of MN, causing damages in excess of $80,000. Although Vonn has never been to NH, she sued Miller in NH federal district court. Which of the following statements is true regarding Vonn's NH lawsuit? (A)The NH federal court lacks personal jurisdiction over the parties because Vonn has insufficient contacts with NH. (B)The NH federal court is an improper venue because the claim did not arise there. (C)The NH federal court may exercise personal jurisdiction over the parties because Vonn consented to such personal jurisdiction. (D) Vonn waived her claim against Miller by filing it in the wrong forum.

C.

Catherine grew up in Kentucky, where she lived for some time. While there, Catherine purchased a stepladder from Hindley's Hardware, a local store. Fleeing an unhappy love life, Catherine moved to Tennessee. After the move, she went back to Kentucky for a visit, chatted with the owner of Hindley's about her new life in Tennessee, and bought a chain saw. Back in Tennessee, Catherine was injured while standing on the ladder attempting to trim a tree with the chain saw. The stepladder collapsed and the saw cut her badly. Catherine would like to sue Hindley's in Tennessee. Could Tennessee constitutionally assert jurisdiction over Hindley's? (A) Yes. Hindley's could foresee that its customer might take the stepladder to another state, including Tennessee. (B) Yes. Hindley's knowingly sold the saw to a resident of Tennessee. (C) No. Hindley's did not seek to serve residents of Tennessee. (D) No. There is not enough profit in a saw and a stepladder to justify jurisdiction over Hindley's.

C. Catherine was the consumer of the ladder and saw, and she purchased both in Kentucky from a local business. There is no indication that Hindley's attempted to market its products in Tennessee. When Hindley's sold the ladder, Catherine was a Kentucky resident. When Hindley's sold Catherine the saw, the salesman was aware that Catherine lived in Tennessee, but that awareness of a single sale to a known resident of Tennessee will not be sufficient to support an argument that Hindley's sought to serve the Tennessee market. The potentially defective products were brought into Tennessee by the consumer, not by the seller.

Assume in the previous fact pattern that Ganja is now suing both Anna Lopez and FMC. Assume also that Texas law does not cap any damages. Ganja claims $35,000 worth of damages from Anna Lopez and $40,001 damages from FMC . As to the lawsuit, the court will A.Hear the lawsuit because both parties are diverse and the amount in controversy is met. B. Hear the lawsuit because Anna Lopez is a citizen of Mexico. C. Dismiss the lawsuit because Ganja cannot reach the jurisdictional amount. D. Dismiss the lawsuit because Texas law does not cap any damages.

C. Dismiss the lawsuit because Ganja cannot reach the jurisdictional amount

Rachel Roy, a citizen of California, sues Beyonce, a citizen of Texas, for defamation, in the appropriate U.S. federal district court. The suit alleges that Beyonce gave Rachel the "evil eye", and called her "Becky with the bad hair" after Rachel was spotted leaving Jay-Z's hotel room one night. Rachel is seeking $100,000 in damages. What result? A. The suit will be heard because it was filed in the appropriate U.S. federal court. B.The suit will be heard because defamation is largely governed by federal law. C.The suit will be dismissed. D. The suit will be dismissed because $100,000 far exceeds the damages Rachel Roy suffered.

C. Even though the citizens are diverse you need allegations to prove your case

You represent Floyd Mayweather who was injured in a bar fight. You would like to file the case in federal court in Massachusetts based on diversity jurisdiction. Assume that Mayweather is a citizen of Massachusetts based on his domicile there. Barristers is incorporated in Delaware. It owns and operates eight bars in Massachusetts. It also owns and operates one in Rhode Island(where it also has its corporate headquarters), two in Connecticut, and one in Maine. A federal court would A. Have diversity jurisdiction, even if Barristers' principal place of business is in Massachusetts, because Mayweather is a citizen of Massachusetts and Barristers is a citizen of Delaware. B. Have diversity jurisdiction, because Mayweather is a citizen of Massachusetts and Barristers is a citizen of Rhode Island and Delaware. C. Lack diversity jurisdiction, because both Mayweather and Barristers are citizens of Massachusetts. D. Lack diversity jurisdiction, if the suit is brought in Massachusetts, since Mayweather is domiciled there.

C. lacks diversity jurisdiction, because both Mayweather and Barristers are citizens of Massachusetts.

What is a forum selection clause?

Clause that identifies the court in which disputes arising under the contract must be litigated, serving to prevent personal jurisdiction challenges when the clause is upheld.

Plaintiff, alleging jurisdiction under 28 U.S.C. § 1331 (federal question) has filed a complaint for violation of his civil rights. He alleges in his complaint that Satan has on numerous occasions caused him misery and unwarranted threats, against the will of plaintiff, that Satan has placed deliberate obstacles in his path and has caused his downfall. A court is likely to: (A). Dismiss for insufficient service of process (B). Dismiss for lack of personal jurisdiction (C). Dismiss for failure to state a claim upon which relief can be granted (D). All of the above

D

Notorious B.I.G. and Faith Evans separated while they were residing in New Jersey. Notorious B.I.G. alleges that Faith Evans shagged Tupac Shakur. Faith denies this, and states that she and Tupac are "just friends". A few months later Faith moves to LA with their daughter. Notorious B.I.G. decides to visit LA to teach Tupac a lesson. While in LA, he stops briefly at Faith's house to visit his daughter. While visiting, Faith serves him with process in a California suit for divorce. Notorious B.I.G. never goes back to Cali again. He files a motion to dismiss for lack of personal jurisdiction. How should the Court rule?

Court should deny the motion because he was present in California when served. He was not there fraudulently.

Bank of America (BOFA) a Delaware corporation with its principal place of business in New York, suedDonald Trump " the Dapper Don" and Anthony Scaramucci ("the Mooch"), two individuals domiciled in Florida. BOFA's claim against the Dapper Don was for $100, 000 and its claim against the Mooch was for $25,000. Both claims arose out of the same loan transaction. BOFA filed a complaint against the Dapper Don and the Mooch in federal court, asserting only State law claims. Which of the following best describes the status of the federal court's subject matter jurisdiction over both claims? A.The court does not have jurisdiction because there must be complete diversity and the requisite amount in controversy for each claim. B.The court has diversity jurisdiction over the claim against the Dapper Don, but must dismiss the claim against the Mooch. C.The court has diversity jurisdiction over both claimsbecause diversity jurisdiction only requires one claim in excess of $75,000. D. The court can exercise diversity jurisdiction over the claim against the Dapper Don, and supplemental jurisdiction over the claim against the Mooch.

D

Dudley, a Georgia citizen, works in security at an airport in Atlanta, Georgia. While at work, Dudley stops Lucky, a Nevada citizen, who was at the airport for a brief layover during his trip home to Nevada after vacationing in Puerto Rico. During the layover, Dudley confiscated a significant amount of cash from Lucky, who claimed to have won it while gambling at a Puerto Rico casino. Lucky returned home to Nevada without his money, while Dudley filed an affidavit in Georgia as part of an effort to have the money forfeited to the government. Assume that Dudley knew that Lucky was on his way home to Nevada at the time Dudley confiscated the money, and also assume that Dudley knew that Lucky was in Nevada at the time Dudley filed his affidavit.Lucky sues Dudley in Nevada state court, alleging that Dudley's affidavit was knowingly false. Assuming Dudley moves to dismiss the Nevada case on personal jurisdiction grounds, the motion would likely be: A. Denied, because Dudley knew where Lucky lived at the time of the alleged intentional misconduct and was thus purposefully directing conduct to Nevada. B. Denied. Even if Dudley did not know where Lucky lived at the time of the alleged intentional misconduct, Dudley could reasonably foresee that his misconduct at a major national airport could result in harm to travelers in their home states. C. Both A and B are accurate. D. Granted, because Dudley did not purposefully direct any conduct to Nevada.

D

Rose McGowan, a resident of California suffered emotional distress when defendant Jeffrey Epstein forcibly tried to kiss her, at the set of the TV series Charmed, in Hollywood California. Epstein resides in New York, owns a palatial mansion in West Palm Beach Florida and a private Caribbean Island in the Virgin Islands. When Epstein, while in New York,heard that McGowan was going to sue him,he jumped up one day and cried out "I have formed the intent to make Californiamy permanent home!!!. McGowan has now sued Epstein in the Federal District Court in LA. Assuming the amount in controversy is met, does the court have subject matter jurisdiction? A. No, because Epstein is domiciled in California. B. No, because Epstein is domiciled in New York. C. Yes, because Epstein is domiciled in New York, Florida and Virgin Islands. D. Yes, because there is complete diversity and the amount in controversy is met.

D

Ganja was born in New York City. When he turned 18, he enlisted in the U.S Army where he was stationed primarily at U.S. military base in San Antonio Texas. While Ganja was stationed in San Antonio, he had vague intentions about what he would doafter completing his army stint. He intended to go to college after his four year stint was complete, though he wasn't sure where he would go to school. In his second year in the army, he was injured and honorably discharged. While driving on Interstate 10 in Texas, Ganjawas hit by car driven by Anna Lopez, born and raised in Mexico city. Anna's tire blew out while she was driving her Ford F-150 truck. Ganja has now sued Ford Motor Company("FMC")in Federal District Court in San Antonio. Ford Motor Company's headquarters is in Detroit Michigan and is also incorporated in Texas.In the lawsuit, Ganja's experts will testify that the blow-out was caused bya faulty tire placed on the truckby FMC. Ganja seeks $100,000 in damages. Assume that Texas law caps damages from motor vehicle injuries to $50, 000. The court will A. Hear the lawsuitbecause Ganja and FMCare diverse,and he has alleged $100,000 in damages. B. Hear the lawsuit because Ganja and Anna Lopez are diverse,and he has alleged $100, 000 in damages. C. Dismiss the suit becauseGanja is a citizen of Texas, thus not diverse from FMC. D. Dismiss the suit because it appears to a legal certainty that Ganja cannot reach the jurisdictional amount.

D

Georgia citizen filed a civil action against a Florida citizen in a Florida state court. The action arose from events that took place in Alabama. Alabama has only one federal district court (the District of Alabama). Florida has three, the Northern, Middle,Southern Districtsof Florida. The Florida citizen resides in the Southern District of Florida, while the state court action filed by the Georgia citizen is pending in a court located in the Northern District of Florida. If the Florida citizen wishes to remove the action to federal district court, in which federal district should the Florida citizen file a notice of removal? A. Either the Southern District of Florida, the Northern District of Florida, or the Districtof Alabama. B. Either the Southern District of Florida or the District of Alabama. C. Either the Southern District of Florida or the Northern District of Florida. D. The Northern District of Florida only.

D

Honey Boo Boo from Georgia sues Mama June, from Florida after she is fired from the set of the show "Here Comes Honey Boo Boo." She sues in Georgia state court seeking $200,000for breach of contract. Three months later, Honey Boo Boo amends the complaint to add a claim under the American with Disabilities Act (ADA), a federal statute, claiming Mama June failed to make reasonable accommodations fora disability that interfered with her ability to speak English, for example,she can no longer say, "a dolla make me hola!Two weeks after receiving the amended complaint, Mama June removes the case to federal court. Which of the following is correct?A.Removal is proper under 28 U.S.C. § 1446 after the amendmentto add the federal claim. B. Removal is proper under Mottley well-pleaded complaint rule, because Honey Boo Boo seeks reliefunderfederal law. C. Removal is not proper, because three months have gone by since the case was filed in state court. D. Removal is not proper because the case could have been removed as originally filed.

D

Kanye and Kim K are married and living in LA, California. Kanye applies to Barry Law school in Orlando, FL. He is eager to attend, but Kim hates the South and is dead-set against leaving LA. They have a big argument, Kanye packs his bags, leaves for Florida, and enrolls at Barry Law. He signs a three-year apartment lease near Disney World, changes the registration on his Hummer, registers to vote in FL, and has all his mail forwarded to his Orlando address. Kim remains in LA. Three months later, Kanye files for divorce in the United States District Court for the Middle District of Florida alleging irreconcilable differences. Kanye's divorce petition should be: A. Heard by the district court, on the basis of federal question. B. Heard by the district court, under diversity jurisdiction. C. Dismissed by the district court, because Kim K and Kanye are still married and so cannot be citizens of different states. D. Dismissed by the district court, under the domestic relations exception.

D

Olive brings an action against Popeye in a Rhode Island trial court. The cause of action is based on an automobile accident that occurred while both Olive and Popeye were vacationing in Maine. Popeye has never been in Rhode Island, owns no property in Rhode Island, and has no contacts with Rhode Island other than the accident with Olive. Popeye appears and makes a timely objection to lack of subject matter jurisdiction, which is denied by the judge. A trial is held and judgment is entered against Popeye for $97,000. Popeye refuses to pay. Popeye now lives in Massachusetts. Olive then brings an enforcement action in a Massachusetts trial court. Popeye appears and moves to dismiss the action on the ground that the trial court in Rhode Island lacked personal jurisdiction over Popeye, and therefore, the judgment is invalid. What result? A. The motion should be granted because Popeye did not purposefully avail himself of the benefits and protections of Rhode Island law. B. The motion should be granted because Popeye has no contacts with Rhode Island. C. The motion should be granted because the auto accident occurred while they were vacationing in Maine. D. The motion should be denied because Popeye appeared in the Rhode Island action and failed to timely object.

D

Pam was born and grew up in Pennsylvania. She went away to college in Massachusetts (her parents paid her tuition), but she always returned home to work in the summers. During her senior year, her parents moved to Virginia, and she lived there between college and law school. She is now a law student at a law school in Texas. She really likes it there and has decided to take the Texas bar exam and look for a job there after graduation. Where is Pam domiciled? (A) Pennsylvania. This is her domicile of origin and she keeps it until she is no longer a student. (B) Massachusetts. She changed her domicile when she went to live there to attend college. (C) Virginia. Her domicile changed when her parents moved there and she went to join them. (D) Texas. By being present there and having the intention to stay indefinitely, she has made the state her domicile.

D

Plaintiff from Kansassues her employer, a Tennessee corporation, for employment discrimination under state law. Plaintiff sues in a Kansas state court and credibly alleges $300,000 in damages. Defendant is served with the summons and complaint on November 1. On December 15, Plaintiff amends her complaint to add a federal employment discrimination claim against Defendant. On December 24, Defendant files a notice of removal in the proper federal district court. Is Defendant's notice of removal timely filed? A. Yes, because Defendantfiled the notice of removal 9 days after he received Plaintiff'samended complaint. B. No, because the case was removable as filed and Defendanthad to file the notice of removal by December 15. C. Yes,because the case was removable as filed and Defendantfiled the notice of removal within one year. D. No, because the case was removable as filed and Defendanthad to file the notice of removal within 30 days.

D

Tony Parker and Eva Longoria were married, but both were cheating on each other. They later divorced. Tony moved to San Antonio Texas and Eva moved to Miami Florida. After a number of years, Eva felt that Tony had cheated her out of a business deal and wanted to sue him. Eva writes to Tony telling him that she is ill and depressed, that she is quitting "Desperate Housewives" her reality TV gig, and would like to discuss some matters with Tony. Tony had never been to Florida and had no need to go there. However, based on the seriousness of Eva's letter, Tony decides to Fly to Miami. When he lands at the airport he see's Eva waiting for him. She does not look ill or depressed. Eva is accompanied by a sheriff, who immediately serves Tony with summons for Eva's lawsuit. Tony Flies back to Texas enraged. Tony hires Counsel who files a motion to dismiss on the basis of lack of personal jurisdiction. The court should A. Grant the motion because a court with jurisdiction over a particular location may exercise in personam jursidiction over a person who resides, maintains connections, or is served notice of legal proceedings in that location. B. Deny the motion because a court with jurisdiction over a particular location may exercise in personam jursidiction over a person who resides, maintains connections, or is served notice of legal proceedings in that location. C. Deny the motion because If the defendant is served with the process while physically in the state, or if s/he is domiciled in the state, general jurisdiction exists. D. Grant the motion because a Defendant cannot be fraudulently induced to enter the state so that the court can have in personam jurisdiction.

D

Tony Parker and Eva Longoria were married, but both were cheating on each other. They later divorced. Tony moved to San Antonio Texas and Eva moved to Miami Florida. After a number of years, Eva felt that Tony had cheated her out of a business deal and wanted to sue him. Eva writes to Tony telling him that she is ill and depressed, that she is quitting "Desperate Housewives" her reality TV gig, and would like to discuss some matters with Tony. Tony had never been to Florida and had no need to go there. However, based on the seriousness of Eva's letter, Tony decides to Fly to Miami. When he lands at the airport he see's Eva waiting for him. She does not look ill or depressed. Eva is accompanied by a sheriff, who immediately serves Tony with summons for Eva's lawsuit. Tony Flies back to Texas enraged. Tony hires Counsel who files a motion to dismiss on the basis of lack of personal jurisdiction. The court should: A. Grant the motion because a court with jurisdiction over a particular location may exercise in personam jursidiction over a person who resides, maintains connections, or is served notice of legal proceedings in that location. B. Deny the motion because a court with jurisdiction over a particular location may exercise in personam jursidiction over a person who resides, maintains connections, or is served notice of legal proceedings in that location. C. Deny the motion because If the defendant is served with the process while physically in the state, or if s/he is domiciled in the state, general jurisdiction exists. D. Grant the motion because a Defendant cannot be fraudulently induced to enter the state so that the court can have in personam jurisdiction

D

Two plaintiffs sued three defendants in federal district court. Plaintiff 1 is a citizen of South Carolina and Plaintiff 2 is a citizen of Florida. Defendant 1 is acitizen of North Carolina, Defendant 2 is a citizen of Georgia, and Defendant 3 is also a citizen of Georgia. After the lawsuit begins, and additional defendant (Defendant 4) who is a citizen of Florida, was added to the lawsuit. Assuming the amount in controversy is greater than $75,000, does the district court have subject matter jurisdiction over the case? A. Yes,because the plaintiffs are diverse from one another. B. Yes, because the plaintiffs were diverse from the original three defendants at the time the original action was filed. C. No, because all the defendants are not diverse from one another. D. No, because the plaintiffs were no longer diverse from all the defendants after the addition of Defendant 4.

D

DeFarge also sells another product, the Super Chopper. This is a large, expensive machine used in manufacturing plants. It advertised the Super Chopper in the January issue of a nationally-circulated trade publication. After a DeFarge-employed salesman made a persuasive presentation on a swing through Maine, DeFarge sold one Super Chopper to a customer there. Because of a flaw in the chopper's safety system, it cut off the hand of Jarvis Lorry, an employee of the customer. Lorry wants to sue DeFarge in Maine for his personal injuries. Can a Maine court constitutionally assert jurisdiction over Lorry's claim against DeFarge? (A) No. This isolated transaction is not enough to be the basis for personal jurisdiction. (B) Yes. DeFarge reached out to Maine because national advertising is sent there. (C) No. DeFarge's customer could sue in Maine, but Lorry cannot. (D) Yes. Lorry's cause of action arises directly out of DeFarge's contacts with Maine, which were purposeful and from which DeFarge earned profits

D DeFarge sent an employee to Maine to sell the Super Chopper is "purposeful availment". That same machine is now said to have injured the plaintiff in Maine. That combination of facts may be sufficient to support a conclusion that DeFarge has sought to serve the Maine market, and that its contacts are sufficient to support personal jurisdiction. Although there is only a single sale, it was purposeful, it was initiated by the defendant's seeking to sell in Maine, and it is precisely this contact that is alleged to have caused damage to the plaintiff

In specific jurisdiction cases, the Court has sometimes articulated another step in the analysis in which the interests of the parties, the forum state, and other states are balanced and utilized these factors as part of the International Shoe inquiry into "traditional notions of fair play and substantial justice." Which of the following factors has been considered in doing the due process fairness analysis? (A) whether the forum state's procedural rules resemble the Federal Rules of Civil Procedure. (B) whether the forum state has good judges. (C) how sincere the plaintiff seems in explaining his desire to sue there. (D) the burden on the defendant of defending itself in the forum state.

D The Court articulated a two-step analysis. The question of minimum contacts/purposeful availment is step one. Step two is where the fair play and substantial justice come in. The Court has listed factors that should be considered: (1) the burden on the defendant; (2) the forum state's interest in adjudicating the dispute; (3) the plaintiff's interest in obtaining convenient and effective relief; (4) the interstate judicial system's interest in obtaining the most efficient resolution of controversies; and (5) the shared interest of the several states in furthering fundamental substantive social policies.

In Pennoyer, Mitchell sued Neff in the Oregon state court to recover fees for the work that Mitchell had performed for Neff in Oregon. After Neff defaulted, Mitchell asked the court to order the sale of Neff's Oregon real estate in order to satisfy the default judgment. Why did the Supreme Court conclude that the Oregon state court did not have jurisdiction to order the sale of Mitchell's Oregon property? a. Neff was not notified in person about the pending lawsuit. b. Neff was not within the state at the time that the suit began. c. A and B both had to be satisfied and were not satisfied in this case. d. The court failed to attach Neff's property before it exercised jurisdiction to resolve Mitchell's claim.

D is the best answer—before a court can exercise in rem jurisdiction, it must first attach the defendant's property. That did not happen here.

Imagine now that Ohio passes a statute that says that "a defendant is subject to personal jurisdiction in Ohio only if the defendant resides in Ohio." Assume that Ohio repeals its enumerated act long arm statute and repeals all other possible bases for personal jurisdiction. Now imagine that Davis, a Wisconsin citizen, visits Ohio, gets drunk at a bar, and recklessly collides with Percy, an Ohio citizen, while in Ohio. Percy suffers serious injuries and sues Davis in Ohio state court. Davis is served at his home in Wisconsin. Davis immediately moves to dismiss the case on personal jurisdiction grounds. The court should A. grant the motion because the exercise of personal jurisdiction would be unconstitutional. B. deny the motion because the long arm statute is unconstitutionally narrow. C. deny the motion because personal jurisdiction is constitutional in this case. D. grant the motion because personal jurisdiction is not authorized by the long arm statute. E. both A and D are correct.

D, therefore, is the answer. The court cannot exercise personal jurisdiction because the statute does not authorize it

Hot Rods is partnership that sells modified classic old cars known as street rods. One of its customers is Custom Rides corporation, a business that is incorporated in Delaware with its principal place of business in Houston Texas. For non payment on a delinquent invoice over $100,000, Hot Rods sued Custom Rides in Iowa Federal Court alleging breach of contract. During trial, a witness testified for the first time that one of the general partners in Hot Rods is a retired Navy Seal who is domiciled in Dallas Texas. Custom Rides has now moved to dismiss the case on jurisdictional grounds. The court will A. Not dismiss the case because lack of jurisdiction is a defense that must be asserted early in the litigation through a pre-answer motion to dismiss B. Not dismiss the case because once the trial commences, it is in the best interest of justice to allow the case to proceed C. Dismiss the case because lack of subject matter jurisdiction may be asserted by a pre-answer motion to dismiss D. Dismiss the case because there is no diversity

D. SMJ NEVER DIES

Pam was born and grew up in Pennsylvania. She went away to college in Massachusetts (her parents paid her tuition), but she always returned home to work in the summers. During her senior year, her parents moved to Virginia, and she lived there between college and law school. She is now a law student at law school in Texas. She really likes it there and has decided to take the Texas bar exam and look for a job there after graduation. Where is Pam domiciled? A: Pennsylvania. This is her domicile of origin and she keeps it until she is no longer a student. B: Massachusetts. She changed her domicile when she went to live there to attend college. C: Virginia. Her domicile changed when her parents moved there and she went to join them. D: Texas. By being present there and having the intention to stay indefinitely, she has made the state her domicile.

D. A person when born acquires a domicile of origin (the parents domicile). In order to change to a new domicile one must intend to make the new place home for an indefinite period of time and must be physically present there at least briefly. Pam has changed her domicile to Texas because she is a resident there and intends to make it her home. The fact that she has not yet secured a permanent job does not prevent Texas from being her domicile.

Ganja was born in New York City. When he turned 18, he enlisted in the U.S Army where he was stationed primarily at U.S. military base in San Antonio Texas. While Ganja was stationed in San Antonio, he had vague intentions about what he would do after completing his army stint. He intended to go to college after his four year stint was complete, though he wasn't sure where he would go to school. In his second year in the army, he was injured and honorably discharged. While driving on Interstate 10 in Texas, Ganja was hit by car driven by Anna Lopez, born and raised in Mexico city. Anna's tire blew out while she was driving her Ford F-150 truck. Ganja has now sued Ford Motor Company("FMC")in Federal District Court in San Antonio. Ford Motor Company's headquarters is in Detroit Michigan and is also incorporated in Texas.In the lawsuit, Ganja's experts will testify that the blow-out was caused by a faulty tire placed on the truck by FMC. Ganja seeks $100,000 in damages. Assume that Texas law caps damages from motor vehicle injuries to $50, 000. The court will A. Hear the lawsuit because Ganja and FMCare diverse, and he has alleged $100,000 in damages. B. Hear the lawsuit because Ganja and Anna Lopez are diverse, and he has alleged $100, 000 in damages. C. Dismiss the suit becauseGanja is a citizen of Texas, thus not diverse from FMC. D. Dismiss the suit because it appears to a legal certainty that Ganja cannot reach the jurisdictional amount.

D. Dismiss the suit because it appears to a legal certainty that Ganja cannot reach the jurisdictional amount

Instead of bringing her lawsuit in California federal court, Paula files suit in Illinois state court. In this case, however, Paula alleges damages of only $50,000. Assuming the Illinois long arm statute reaches as far as the Constitution allows and David makes all relevant motions to dismiss, the court A. cannot hear the case because it lacks subject matter jurisdiction. B. cannot hear the case because it lacks personal jurisdiction; even though David is from Illinois, Paula now lives in California and no longer has any contacts with Illinois. C. cannot hear the case because it lacks both personal and subject matter jurisdiction. D. can hear the case because it has subject matter jurisdiction and personal jurisdiction.

D. In this case, not only did the claim arise out of David's contacts with Illinois, but David lives in Illinois

Plaintiff from Kansas sues her employer, a Tennessee corporation, for employment discrimination under state law. Plaintiff sues in a Kansas state court and credibly alleges $300,000 in damages. Defendant is served with the summons and complaint on November 1. On December 15, Plaintiff amends her complaint to add a federal employment discrimination claim against Defendant. On December 24, Defendant files a notice of removal in the proper federal district court. Is Defendant's notice of removal timely filed? A. Yes, because Defendant filed the notice of removal 9 days after he received Plaintiff's amended complaint. B. No, because the case was removable as filed and Defendant had to file the notice of removal by December 15. C. Yes, because the case was removable as filed and Defendant filed the notice of removal within one year. D. No, because the case was removable as filed and Defendant had to file the notice of removal within 30 days.

D. No, because the case was removable as filed and Defendant had to file the notice of removal within 30 days

Two plaintiffs sued three defendants in federal district court. Plaintiff 1 is a citizen of South Carolina and Plaintiff 2 is a citizen of Florida. Defendant 1 is a citizen of North Carolina, Defendant 2 is a citizen of Georgia, and Defendant 3 is also a citizen of Georgia. After the lawsuit begins, and additional defendant (Defendant 4) who is a citizen of Florida, was added to the lawsuit. Assuming the amount in controversy is greater than $75,000, does the district court have subject matter jurisdiction over the case? A. Yes, because the plaintiffs are diverse from one another. B. Yes, because the plaintiffs were diverse from the original three defendants at the time the original action was filed. C. No, because all the defendants are not diverse from one another. D. No, because the plaintiffs were no longer diverse from all the defendants after the addition of Defendant 4.

D. No, because the plaintiffs were no longer diverse from all the defendants after the addition of Defendant 4

Honey Boo Boo from Georgia sues Mama June, from Florida after she is fired from the set of the show "Here Comes Honey Boo Boo." She sues in Georgia state court seeking $200,000for breach of contract. Three months later, Honey Boo Boo amends the complaint to add a claim under the American with Disabilities Act (ADA), a federal statute, claiming Mama June failed to make reasonable accommodations fora disability that interfered with her ability to speak English, for example, she can no longer say, "a dolla make me hola!Two weeks after receiving the amended complaint, Mama June removes the case to federal court. Which of the following is correct? A.Removal is proper under 28 U.S.C. § 1446 after the amendment to add the federal claim. B. Removal is proper under Mottley well-pleaded complaint rule, because Honey Boo Boo seeks relief under federal law. C. Removal is not proper, because three months have gone by since the case was filed in state court. D. Removal is not proper because the case could have been removed as originally filed.

D. Removal is not proper because the case could have been removed as originally filed

A Georgia citizen filed a civil action against a Florida citizen in a Florida state court. The action arose from events that took place in Alabama. Alabama has only one federal district court (the District of Alabama). Florida has three, the Northern, Middle,Southern Districts of Florida. The Florida citizen resides in the Southern District of Florida, while the state court action filed by the Georgia citizen is pending in a court located in the Northern District of Florida. If the Florida citizen wishes to remove the action to federal district court, in which federal district should the Florida citizen file a notice of removal? A. Either the Southern District of Florida, the Northern District of Florida, or the District of Alabama. B. Either the Southern District of Florida or the District of Alabama. C. Either the Southern District of Florida or the Northern District of Florida. D. The Northern District of Florida only.

D. The Northern District of Florida

Bank of America (BOFA) a Delaware corporation with its principal place of business in New York, suedDonald Trump " the Dapper Don" and Anthony Scaramucci ("the Mooch"), two individuals domiciled in Florida. BOFA's claim against the Dapper Don was for $100, 000 and its claim against the Mooch was for $25,000. Both claims arose out of the same loan transaction. BOFA filed a complaint against the Dapper Don and the Mooch in federal court, asserting only State law claims. Which of the following best describes the status of the federal court's subject matter jurisdiction over both claims? A.The court does not have jurisdiction because there must be complete diversity and the requisite amount in controversy for each claim. B.The court has diversity jurisdiction over the claim against the Dapper Don, but must dismiss the claim against the Mooch. C.The court has diversity jurisdiction over both claims because diversity jurisdiction only requires one claim in excess of $75,000. D. The court can exercise diversity jurisdiction over the claim against the Dapper Don, and supplemental jurisdiction over the claim against the Mooch.

D. The court can exercise diversity jurisdiction over the claim against the Dapper Don, and supplemental jurisdiction over the claim against the Mooch

Rose McGowan, a resident of California suffered emotional distress when defendant Jeffrey Epstein forcibly tried to kiss her, at the set of the TV series Charmed, in Hollywood California. Epstein resides in New York, owns a palatial mansion in West Palm Beach Florida and a private Caribbean Island in the Virgin Islands. When Epstein, while in New York, heard that McGowan was going to sue him, he jumped up one day and cried out "I have formed the intent to make California my permanent home!!!. McGowan has now sued Epstein in the Federal District Court in LA. Assuming the amount in controversy is met, does the court have subject matter jurisdiction? A. No, because Epstein is domiciled in California. B. No, because Epstein is domiciled in New York. C. Yes, because Epstein is domiciled in New York, Florida and Virgin Islands. D. Yes, because there is complete diversity and the amount in controversy is met.

D. Yes, because there is complete diversity and the amount in controversy is met

Imagine that you are a New Mexico citizen and that while traveling in Arizona, you enter a local hardware store looking for a chainsaw. You tell the owner that you intend to bring the chainsaw back to New Mexico, and the owner then sells you the chainsaw. You return to New Mexico, are injured by the chainsaw, and file a lawsuit in New Mexico against the hardware store. Would it be constitutional for the New Mexico court to exercise personal jurisdiction over the store? A. Yes. Because the customer told the store owner where the chainsaw was being taken, the owner directed a contact to New Mexico. B. Yes. Regardless of whether the owner knew where the chainsaw was being taken, the owner should have foreseen that a small product like a chainsaw could be used in a neighboring state. C. No. Although the claim arose out of the defendant's contact with the state, personal jurisdiction would be unfair and unreasonable in this case. D. No. The claim does not arise out of the defendant's contacts with New Mexico. E. Yes. Although the store's contact with New Mexico is indirect, the reasonableness factors overcome the weak contacts that exist.

D. There would not be personal jurisdiction here. The defendant did not direct the product to New Mexico, and the defendant did not otherwise avail itself of the privilege of doing business in the state.

Snoop Dogg (a citizen of California) plans to sue Defendant Ford Motor Co., (incorporated in Delaware with its principal place of business in Michigan) for injuries suffered in an auto accident. Snoop believes that problems with the Ford F-350 series vehicle caused the accident. If Snoop sues Ford in California state court seeking $200,000 damages, can Ford remove the case to federal court? A. No, because the well-pleaded complaint rule indicates that a plaintiff is the master of his complaint. B. Yes, because defendants can always remove from state court to federal court. C. No, because no federal question is involved. D. Yes, because the federal court would have had original jurisdiction over Snoop's claim had Snoop filed there originally.

D. Yes, because the federal court would have had original jurisdiction over Snoop's claim had Snoop filed there originally

What is implied consent?

Driving through the state or failing to object to personal jurisdiction.

What arised out of the International Shoe case

Due process requires only that in order to subject a defendant to judgement in personam, if he be not present with the territory of the forum, he have certain minimum contacts with it such that the maintenance of the suit does not offend "traditional notions of fair play and substantial justice". Overturning Pennoyer.

Trellis Industries is a Delaware corporation with its principal place of business in Delaware. Trellis makes laptop batteries, which are used as components in computers manufactured by Nectarine Computer, Inc., a Delaware corporation with its principal place of business in Massachusetts. Trellis ships its batteries to Nectarine's factory in Massachusetts, knowing that Nectarine incorporates the batteries into its computers for thousands of sales in every United States jurisdiction. Trellis does not send its batteries to any other company; it does not deal directly with Nectarine's customers; and it engages in no other business-related activities outside of the state of Delaware. Eugene (an Idaho citizen) buys a computer from Nectarine, which is shipped to him from Nectarine's plant in Massachusetts. The computer catches on fire, causing Eugene to lose all of his valuable data. Eugene sues Nectarine and Trellis in Idaho state court, alleging that the fire was due to a defective Trellis battery. Trellis moves to dismiss for lack of personal jurisdiction. Assuming Idaho's long arm statute reaches as far as the Constitution allows, which of the following statements most accurately describes what the United States Supreme Court has held in this type of case? A. The Idaho state court has personal jurisdiction over Trellis. B. The Idaho state court would not have personal jurisdiction over Trellis, because Trellis lacks sufficient contacts with Idaho. C. The Idaho state court would not have personal jurisdiction because Trellis lacks sufficient contacts with Idaho and because the exercise of personal jurisdiction would be unfair and unreasonable. D. The United States Supreme Court has left unresolved whether the Idaho state court would have personal jurisdiction over Trellis. E. The United States Supreme Court has left unresolved whether Trellis has sufficient contacts with Idaho, but the Supreme Court has held that the exercise of personal jurisdiction would be unfair in this type of case.

D—the Court has left this issue largely unresolved.

Given that the Court was willing to assume that Daimler had contacts in California through MBUSA, why were those contacts insufficient for specific jurisdiction?

Even if Daimler is considered to have contacts in California through its subsidiary, the plaintiffs' claims did not arise out of those contacts under two commonly used tests.

What is the relationship between corporations and general jurisdiction?

General jurisdiction exists only if the defendant has its principal place of business in the state, or if the corporation is incorporated in the state, or if the corporation carries on a continuous and systematic part of its business in the state. aka, the corporation is "essentially at home in the forum state".

In World-Wide Volkswagen, the consumer took the car from New York to Oklahoma, and the Court concluded that the seller of the car was not subject to personal jurisdiction in Oklahoma. How is the theory of personal jurisdiction that the Court rejected in World-Wide Volkswagen different from the "stream of commerce" theory?

In World-Wide Volkswagen, the product entered the forum state (Oklahoma) as a result of the consumer's conduct.

Where would Susan be domiciled in the following: Susan Gordon goes from Pennsylvania to Ricks College, planning to practice nursing after she finishes her degree, but with no plan as to where she will do so. A month after starting school, she sues the Pennsylvania defendants.

Here, Gordon has no clear intent to leave Idaho. She might, but has no definite plan to do so or to do anything else. The better answer is that she acquires domicile in Idaho.

Everett, a passenger in Lipmann's car, is injured when Lipmann's car collides with Ritter's. She sustains $100,000 in injuries and sues both Lipmann and Ritter for negligence. Under applicable tort law, whichever defendant is found negligent would be fully liable for her injuries. If both are found negligent, both would be liable to pay her full damages. (Of course, if she collected $100,000 from Lipmann she could not then collect another $100,000 from Ritter, but she could demand payment of the full amount from either defendant.) Does she meet the amount-in-controversy requirement?

Here, either defendant may end up incurring a judgment for more than $75,000. If the jury finds that the accident was Lipmann's fault, Lipmann will be liable for $100,000. If they find that it was Ritter's fault, Ritter will be liable for $100,000. If they find it was due to the negligence of both defendants, they may (depending on local law) both be liable for $100,000. So either might be liable for more than $75,000. Of course, either might escape liability entirely. Or, both might escape liability, if the jury finds that neither was negligent. But either might be liable for more than the jurisdictional amount, which is all that is required. The same result would follow in a case in which the plaintiff sues two defendants in the alternative. If Cornwell is hit by a car but is unsure whether it was driven by Jeckle or Hyde, she might sue them both for injuries worth $120,000. Either might be liable (if found to be the driver) but both will not be liable, only the defendant who was driving. Cornwell meets the amount requirement against each.

Where would Susan be domiciled in the following: Susan Gordon goes from Pennsylvania to Ricks College, planning to get her degree and practice nursing in California. After three months at college, she decides she likes Idaho and will stay and practice nursing there after she completes her degree. She brings suit a month after she makes that decision.

Idaho. Gordon did not acquire domicile when she moved to Idaho but did when her intent changed while she was living there. The statement quoted in Gordon that "[i]t is the intention at the time of arrival which is important" is misleading. Even though Gordon did not form a domicile on the day she moved there, she does form it later when she is living there and decides to stay

Where would Susan be domiciled in the following: Susan Gordon goes from Pennsylvania to Ricks College, planning to get her degree and practice nursing in Idaho. After three months at college, she decides that she does not like Idaho and will return to Pennsylvania after she completes her degree. The next week she brings the suit.

Idaho. When Gordon arrived in Idaho planning to stay, she acquired domicile there. Although she now intends to leave Idaho, she won't lose her Idaho domicile until she goes to another state with the intent to remain there indefinitely

Imagine that Lowell Franklin purchased an insurance policy from the International Life Insurance Company while living in North Carolina and that he moved to California only a week before his death. Assume that the beneficiary of the policy, Lulu McGee, had always lived in California. If the insurance company had no other connection to California, would a California court have authority to exercise personal jurisdiction over the insurance company if the company makes a timely objection? A. No. The California statute does not confer personal jurisdiction in this case. B. No. Personal jurisdiction in this case would probably be unconstitutional. C. Yes. The California statute permits personal jurisdiction in this type of case, because the case involves an insurance policy that covered someone who was a California citizen at the time of death. D. Yes. The statute probably applies here, and its application to this case would be constitutional: Witnesses regarding the cause of McGee's death are in California, and California has an interest in ensuring that one of its citizens recovers on an insurance policy. In addition, Mrs. McGee has an interest in suing in California.

In the end, California may be a reasonable place to hear this case given that important witnesses live there and the beneficiary of the policy lives there. But without the defendant's purposeful contacts in California, there is probably no personal jurisdiction. Thus, B is the best answer.

Holmes Test: does this arise under federal law? Jones, driving down Interstate 95, a part of the federal interstate highway system, collides with Smith. Jones sues Smith for negligence.

Jones has sued for negligence. It is negligence law—which is state tort law—that creates his right to sue. Although the accident took place on a federal highway, Jones does not seek relief under federal law. If there were a Federal Recovery for Accidents on Interstate Highways Act, authorizing injured parties to sue for such accidents, Jones could sue under that Act and invoke 28 U.S.C. § 1331. But there isn't, and Jones didn't. He sued for negligence, a state law claim. No jurisdiction.

What is Quasi in Rem jurisidiction?

Means the power of the court to render and enforce its judgment against property under the court's power and control but which property is not subject of the suit. The judgment does not bind D personally, and cannot be enforced against any other property owned by D except that under the court's power and control.

What is In Rem jurisdiction?

Means the power of the court to render judgment against the property that is subject of the suit. Jurisdiction limited to situations where the property is located within the physical borders of the state and where it is necessary for the state to be able to bind all persons regarding the property's ownership and use.

Consider whether the amount requirement is met in the cases below. Assume that the parties are diverse and that the amounts demanded are supportable on the evidence. P1 sues D for $50,000. P2 joins as a coplaintiff, asserting a claim against D for $60,000 in the action, arising from the same business dispute.

No good. Neither plaintiff meets the amount requirement independently. The plaintiffs cannot add their claims together to reach the $75,000 plus threshold.

In which of the following cases is the amount-in-controversy requirement met as to all claims? Lopez sues Alou for $50,000 for losses he suffered in a business deal. He also sues Antoine, as a codefendant in the same action, for $60,000 in losses Antoine caused in the same deal.

No, courts have not allowed a plaintiff to aggregate claims against separate defendants, as Lopez has tried to do

In which of the following cases is the amount-in-controversy requirement met as to all claims? Adams sues Bickel for $60,000 for injuries in an auto accident. Rajiv joins as a coplaintiff, claiming $50,000 from Bickel for his injuries in the same accident.

No, courts have not allowed separate plaintiffs, each with an insufficient claim, to join together in a single action and aggregate their claims to meet the requirement. So, neither Adams nor Rajiv meets the amount requirement.

Merkle sues Rico for personal injuries she suffered in an auto accident with Rico in June 2016. Could she later sue Rico in a new suit for damage to her car in the same accident?

No, since both the injury and the damage arose out of the same occurrence. Since she already sued for this accident she is not entitled to a second "bite at the apple"

Holmes Test: does this arise under federal law? Apex Company has a patent on inverse rototurnbuckles. (Patents are granted by the federal Patent Office under a federal statute.) Allied Manufacturing wants to use Apex's rototurnbuckles as a component of a threshing machine they manufacture. Apex executes a contract that licenses Allied to do so, provided it pays Apex $10 for each turnbuckle it manufactures. Allied falls behind in its payments and Apex sues to collect the balance due.

No- under a contract

Holmes Test: does this arise under federal law? Larry Lawstudent borrows $5,000 from Metropolitan Bank, under a federal student loan program. Under the program, banks make loans to students under individual loan agreements with each student. The statute setting up the federal loan program includes a guarantee that, if the student defaults, and the bank cannot collect after diligent efforts, the federal government will assure payment of the outstanding balance. Larry defaults on his loan, and Metropolitan sues him for the amount due.

No. Breach of contract- State case.

Imagine that shortly after Pennoyer was decided, Paul sued Dora in Massachusetts state court and alleged that Dora breached a contract. Dora lived in Connecticut, so Paul hired a process server who located Dora in Connecticut and personally handed her the complaint and summons. Would the Massachusetts state court have had personal jurisdiction over Dora using the standard set out in Pennoyer?

No. In this case, the defendant received notice of the pending lawsuit in a manner clearly likely to inform her that she had been sued. As a result, notice is proper. But under Pennoyer, personal service of process—proper notice—did not suffice to establish personal jurisdiction. A court only had jurisdiction over persons and property located within its territory, not outside of it

Rota, a Pennsylvania citizen, sues Matthews, Bernstein, Rollins and Grey, a 120-person law firm with offices in New York City. Her claim is for fraud. Most of the partners live in New York, but ten live in New Jersey and one in Pennsylvania. Is this a diversity case?

No. The law firm is considered to be a citizen of New York, New Jersey, and Pennsylvania. As a result, there is no complete diversity. Since the claim arises under state law, and there is no diversity jurisdiction, the case must be brought in state court.

Consider whether the amount requirement is met in the cases below. Assume that the parties are diverse and that the amounts demanded are supportable on the evidence. P sues D1 for $120,000 and D2 for $60,000, in a single action arising from a single business dispute.

P meets the amount requirement against D1 but not against D2. She cannot add the two claims together. She is suing D1 for more than the jurisdictional amount, so that claim is proper. But her claim against D2 does not meet the amount requirement and cannot be aggregated with the claim against D1. D2 will have to be dropped from the suit.

What happened in Pennoyer v. Neff?

Pennoyer Rule: A valid in personam judgement requires in-state service of D (presence), or ther voluntary appearance (consent).

Where would Susan be domiciled in the following: Susan Gordon goes to Ricks College, planning to get a two-year nursing degree and then move to California to work in a hospital there. Before leaving for Ricks College, she announces to her friends that she is never coming back to Pennsylvania. Two months after starting school, she sues the Pennsylvania doctors.

Pennsylvania. She has not formed a new domicile in Idaho, since she is only in Idaho for a definite period. She hasn't formed one in California either, because she hasn't moved there yet. Ironically, though she swears she will never set foot in Pennsylvania again, she does not lose her Pennsylvania domicile until she forms a new one.

28 U.S.C. 1367(b)

Plaintiff cannot bring claims against 3rd party defendants if they were joined to the case later on under rule 14, 19, 20, or 24

Arch Technologies, Inc. is incorporated in Oregon, with its principal place of business in California. It sues Sullivan Castings Corporation (incorporated in Delaware with its principal place of business in Oregon) in federal court under a federal statute barring deceptive commercial practices in interstate commerce. It claims losses of $500,000. It also claims that Sullivan's conduct constitutes an unfair business practice under an Oregon statute and seeks punitive damages on the state law claim. Oregon courts have never decided whether punitive damages may be awarded under its business practices statute.The federal judge wants to decline jurisdiction over the unfair business practices claim, since the governing law is unclear and the Oregon courts can provide a "surer footed" reading of the statute. The judge should A. dismiss the state law claim. B. remand the state law claim to the Oregon state courts. C. dismiss the entire case. D. remand the entire case to the Oregon state courts.

Remand to a state court is not an option in a case filed originally in federal court. Nor can the judge dismiss the entire case, as C suggests. Even if it would be more efficient to dismiss the entire case, forcing the plaintiff to litigate all of her claims in state court, the federal court has jurisdiction over the federal claim and generally must hear it. So A is right: The judge should dismiss the state law claim, retaining jurisdiction over the federal claim.

What is the difference between general and specific jurisdiction?

Specific jurisdiction begins with the connection between the controversy and the forum state."General jurisdiction, by contrast, does not begin with a relationship between the controversy and the forum state. Rather, general jurisdiction begins and ends with the relationship between the defendant and the forum state

Recall from earlier chapters that a party can typically raise subject matter jurisdiction for the first time long after the case has begun, even on direct appeal. Why is personal jurisdiction, unlike subject matter jurisdiction, so easily waived?

Subject matter jurisdiction concerns the court's constitutional authority to hear a case. Without that power, the court's decisions can have no legal effect, so the parties have no authority to waive subject matter jurisdiction. In contrast, personal jurisdiction is not a limitation on the court's inherent authority; it is a procedural protection for a party (typically the defendant) under the Fourteenth Amendment Due Process Clause. If a defendant wants to litigate in the state that the plaintiff has chosen, the defendant may waive the procedural safeguard of personal jurisdiction. In other words, personal jurisdiction is a defense that is personal to the defendant, whereas subject matter jurisdiction implicates systemic concerns.

Based on the contacts that International Shoe had with the state of Washington, what type of in personam jurisdiction could have been exercised by a Washington court over International Shoe?

Surely there would be specific jurisdiction. International Shoe had contacts with Washington through the representatives who sold the company's shoes there. Moreover, the lawsuit arose out of unemployment compensation payments that were owed because of the employment of those sales representatives. The Court wrote: "The obligation which is here sued upon arose out of [International Shoe's] very activities [in the forum]. "The Court also noted that International Shoe's activities in Washington were "neither irregular nor casual. They were systematic and continuous throughout the years in question." However, these activities were probably not sufficient to support general jurisdiction. A corporation can have ongoing contacts with a state, but not enough contacts to support general jurisdiction. See, e.g., Daimler AG v. Bauman, 571 U.S. 310 (2014) (making the distinction between having "continuous and systematic contacts" and being "at home" in the forum). Thus International Shoe's limited but ongoing sales in Washington were probably not sufficient to support general jurisdiction in Washington

The Court did not conclusively determine whether Asahi had contacts in California, so why did the Court reverse the California Supreme Court?

The Court ultimately concluded that the reasonableness factors had not been satisfied. Those factors include an examination of "the burden on the defendant, the interests of the forum State, and the plaintiff's interest in obtaining relief. [A court] must also weigh in its determination 'the interstate judicial system's interest in obtaining the most efficient resolution of controversies; and the shared interest of the several States in furthering fundamental substantive social policies.'"

What is personal (in personam) jurisdiction?

The ability of a court to exercise power over a particular defendant or item of property. Personal jurisdiction identifies which federal district court may hear the case. In order to be establish, it must satisfy both constitutional and statutory requirements.

In which of the following cases is the amount-in-controversy requirement met as to all claims? Adams sues Bickel for $40,000 for slander, and for $50,000 for an unrelated breach of contract.

The amount requirement is met, since a plaintiff may add her claims together to reach the $75,000 plus threshold, even if they are unrelated.

Consider whether the amount requirement is met in the cases below. Assume that the parties are diverse and that the amounts demanded are supportable on the evidence. P sues D for $20,000 for an accident claim, $30,000 for libel, and $40,000 for a contract breach.

The amount requirement is met. A single plaintiff suing a single defendant may add together any claims she has against the defendant to meet the amount requirement, even if they are unrelated.

Consider whether the amount requirement is met in the cases below. Assume that the parties are diverse and that the amounts demanded are supportable on the evidence. P sues D1 for $40,000 and D2 for $60,000, in a single action arising from a single business dispute.

The amount requirement is not met. A plaintiff may not add claims against different defendants to meet the amount requirement (absent a common and undivided interest—see note 3 on page 84).

Assume that Mitchell sues Neff in Oregon, alleging that Neff hired Mitchell to perform legal services in Oregon and failed to pay Mitchell the resulting legal fees. Also assume that the suit arose after International Shoe was decided and that Mitchell found Neff in California and served Neff with process in California. Would it be constitutional for the Oregon court to exercise in personam jurisdiction over Neff? A. Yes, because Neff was served personally in California. B. Yes, because Neff had a contact with Oregon, and the claim arose out of that contact. C. No, because Neff was not served within Oregon's borders. D. No, because the claim in Oregon is unrelated to Neff's contact with California.

The answer, then, is B. Neff hired Mitchell in Oregon, and Neff failed to pay Mitchell in Oregon. Mitchell sued Neff for the failure to pay the fees in Oregon. Neff, therefore, had a contact within Oregon, and Mitchell's claim arose out of that contact. Thus, if the suit were brought today, the Oregon court could exercise in personam jurisdiction over Neff.

Which of the following cases may be removed to federal court? A. Martinez, from Texas, sues Murphy, from Utah, on a state law breach of contract claim, in a Utah state court, for $200,000. B. Martinez, from Texas, sues Murphy, from Utah, and Mercer, from Nevada, on a state law breach of contract claim, in a Utah state court, for $200,000. C. Martinez, from Texas, sues Murphy, from Utah, on a claim arising under federal law. She brings the suit in a Utah state court. D. Martinez, from Texas, sues Hawkins, from Texas, on a state law claim for negligence. Hawkins counterclaims against Martinez for violation of a federal statute. (A counterclaim is a claim asserted by a defendant seeking relief from the plaintiff.)

The case described in C is removable, even though there is an in-state defendant, because it arises under federal law. (Section 1441(a) authorizes removal, since the federal court would have original jurisdiction over the case, and the exception in § 1441(b)(2) does not apply.) This makes sense. Federal courts provide an experienced and hospitable forum for application and interpretation of federal law. If either party wants the federal court to hear a case arising under federal law, she should be able to invoke it.

Jiminez, a Colorado plaintiff, sues three defendants, Kramer and Jost (two individuals from Wyoming) and Delta Corporation, incorporated in Wyoming with a good deal of business in Wyoming and Colorado. Just before trial, Delta moves to dismiss, claiming that Colorado is its principal place of business. The court takes evidence and hears argument on the issue and concludes that Delta is right. Consequently, it is a citizen of both Wyoming (based on incorporation) and Colorado (based on its principal place of business). Must the court dismiss the case?

The court cannot hear the case as originally framed because there is not complete diversity. However, it need not dismiss the entire suit; it can order Delta dropped as a defendant, thus "perfecting diversity," and continue with the case against the two individual defendants.

In Holmes v. Sopuch, 639 F.2d 431 (8th Cir. 1981), the plaintiff, Holmes, had been living in Missouri and working for the federal Defense Mapping Agency. The DMA sent him to a one-year program at Ohio State University during the 1978-79 academic year. He and his wife moved to Ohio, where they leased an apartment for one year. In February 1979, while he was still living in Ohio, he brought a diversity action against two Missouri citizens. Holmes testified that "after finishing his studies at Ohio State he would obtain the best position available with the DMA and that he never intended Ohio to be his permanent home. Moreover, there is no DMA facility in Ohio which Holmes might have chosen upon the completion of his studies. Holmes also testified that he might have chosen to return to the St. Louis DMA facility." Did Holmes acquire domicile in Ohio?

The court held that Holmes did not acquire domicile in Ohio during his time there. Although he resided there, he was not there "indefinitely." He intended to leave Ohio at a definite time, since he planned to resume his work for the DMA, and he could only do so by leaving Ohio.

In Kahn v. Hotel Ramada of Nevada, 799 F.2d 199 (5th Cir. 1986), the plaintiff left his suitcase for safekeeping at a hotel, and it was lost. Kahn, a jewelry broker, alleged that it contained some $50,000 in jewelry (the amount at the time had to exceed $10,000). A Nevada statute limited an innkeeper's liability for lost property to $750. The hotel moved to dismiss Kahn's suit for failure to meet the amount-in-controversy requirement. What should the court do?

The court should dismiss the case for lack of subject matter jurisdiction, since the plaintiff cannot meet the amount-in-controversy requirement. Unless the plaintiff has an argument that the statute is somehow inapplicable, there is a legal certainty that the plaintiff cannot recover more than $750, well below the required amount.

Assume under the same facts that Honey Boo Boo brings the suit in the Southern district of Georgia. Andretti files a motion to dismiss for want of personal jurisdiction. Judgment for whom and why?

The court should grant the motion. The issue is if a Georgia court has personal jurisdiction over Andretti. Under the rule of minimum contacts, he has no relationship to the State of Georgia.

Notorious B.I.G. and Faith Evans separated while they were residing in New Jersey. Notorious B.I.G. alleges that Faith Evans shagged Tupac Shakur. Faith denies this, and states that she and Tupac are "just friends". A few months later Faith moves to LA with their daughter. Notorious B.I.G. decides to visit LA to teach Tupac a lesson. While in LA, he stops briefly at Faith's house to visit his daughter. While visiting, Faith serves him with process in a California suit for divorce. Notorious B.I.G. never goes back to Cali again. He files a motion to dismiss for lack of personal jurisdiction. How should the Court rule?

The court should rule that the motion should be denied. At issue is whether the court has personal jurisdiction over the Notorious B.I.G. Under personal jurisdiction, a defendant can be served as long as they are physically present in the forum if not fraudulently for judicial proceedings. Here, Faith served B.I.G while at her home in L.A., while B.I.G. was seeing their daughter on his own accord. The court should rule that the motion should be denied.

What is an enumerated long-arm statute?

The statute enumerates a list of actions by the defendant that will allow the state to assert jurisdiction over it (FL does this)

Penelope buys a hot coffee at a McDonald's in Florida and spills it on herself, causing third-degree burns. Upon returning to her home in Georgia, Penelope sues McDonald's in a Georgia state court for negligence, alleging that the store sold excessively hot coffee and caused her to suffer damages. Assume that the Georgia long arm statute reaches as far as the Constitution allows and that McDonald's is incorporated in Delaware, has its principal place of business in Illinois, and has seventy-five stores in Georgia. If McDonald's moves to dismiss the case on personal jurisdiction grounds, the court will probably A. grant the motion. Even under the most liberal definition of "arising out of," this case does not arise out of any contact that McDonald's had with Georgia. B. deny the motion. The state court has the authority to hear all state law claims as a matter of subject matter jurisdiction. C. grant the motion. McDonald's has its principal place of business and place of incorporation outside of Georgia, so a Georgia court cannot exercise general jurisdiction in Georgia. And for the reasons set out in A, there is no specific jurisdiction. D. deny the motion, because the court would likely have general jurisdiction over McDonald's in Georgia.

Until the Court decided Daimler in early 2014, we believed that D was the probable answer to this question, reasoning that McDonald's had "continuous and systematic" contacts with the forum state through its numerous restaurants there. Although Goodyear had made us more tentative about that answer because of the Court's reference in that case to the "at home" concept, we still thought that the answer was D. Now, however, Daimler makes C the best answer. Daimler makes clear that the "at home" concept means that general jurisdiction typically exists over corporations only where they are incorporated or have their principal place of business.

Does this satisfy the Holmes Test to bring a case in the federal court? Bluestein, a fifty-five-year old employee of Concept Corp., is fired. He claims that he was fired based on his age and replaced by a younger worker. The federal Age Discrimination in Employment Act bars age discrimination in employment and allows actions for damages for it. Bluestein sues for damages under the Act.

Yes

Does this satisfy the Holmes Test to bring a case in the federal court? Zander holds a patent on inverse rototurnbuckles. Federal law provides that a patent holder enjoys the right to exclude others from manufacturing a patented device unless the patent holder grants a license to another to do so. Cassevites, without obtaining a license from Zander, begins to make rototurnbuckles. Zander sues to obtain an injunction barring Cassevites from doing so.

Yes

Holmes Test: does this arise under federal law? Ace Tractor Company ships all its tractors via Great Northern Railroad, the only railroad that serves the area where its factory is located. Great Northern notifies Ace that it intends to raise its rates by 20 percent. Ace sues to enjoin the increase on the ground that the new rates exceed those allowed by the federal Interstate Commerce Commission.

Yes, federal law creating cause of action

Holmes Test: does this arise under federal law? Instead of suing Allied to collect the balance due under the licensing agreement, Apex sues to enjoin Allied from continuing to manufacture the patented rototurnbuckles, arguing that making them without its permission infringes Allied's patent. Context (Apex Company has a patent on inverse rototurnbuckles. (Patents are granted by the federal Patent Office under a federal statute.) Allied Manufacturing wants to use Apex's rototurnbuckles as a component of a threshing machine they manufacture. Apex executes a contract that licenses Allied to do so, provided it pays Apex $10 for each turnbuckle it manufactures. Allied falls behind in its payments and Apex sues to collect the balance due.)

Yes- Patent infringement

Janice hired Bornstein to build a garage next to her house and claims that he built it too small. She also claims that his backhoe damaged her shrubbery in the process. Can she sue on both claims in a single suit?

Yes. A P may assert whatever claims she has against the D, so they may all be settled in a single litigation.

Assume now that Demo's driver got into an accident while driving in Massachusetts. Would there be personal jurisdiction over Demo in Pennsylvania if the accident victim sued Demo there?

Yes. Even though there is probably no specific jurisdiction in Pennsylvania, there is general jurisdiction over Demo in Pennsylvania, because Demo is effectively at home in the state. Thus, both the Massachusetts courts (specific in personam) and Pennsylvania courts (general in personam) would have in personam jurisdiction over Demo in this case.

Merkle sues Rico for personal injuries she suffered in an auto accident with Rico in June 2016. Could Merkle bring a later action against Rico for injuries in a different accident they had in 2015?

Yes. They arose out of different occurrences at different times.

Holmes Test: does this arise under federal law? On the facts of example B, assume that Metropolitan is unable to collect from Larry and demands payment from the federal agency administering the loan program. The federal program administrator refuses payment, and Metropolitan sues the agency to collect. (Disregard the possibility that there is federal jurisdiction due to the fact that the United States is a party to the action.) B states: (Larry Lawstudent borrows $5,000 from Metropolitan Bank, under a federal student loan program. Under the program, banks make loans to students under individual loan agreements with each student. The statute setting up the federal loan program includes a guarantee that, if the student defaults, and the bank cannot collect after diligent efforts, the federal government will assure payment of the outstanding balance. Larry defaults on his loan, and Metropolitan sues him for the amount due.)

Yes. since the bank went to the federal government to collect money after trying to get the student to pay.

Rose McGowan, a resident of California suffered emotional distress when defendant Jeffrey Epstein forcibly tried to kiss her, at the set of the TV series Charmed, in Hollywood California. Epstein resides in New York, owns a palatial mansion in West Palm Beach Florida and a private Caribbean Island in the Virgin Islands. When Epstein heard that McGowan was going to sue him in California, he jumped up one day and cried out " I have formed the intent to make this state my permanent home!!!. McGowan has now sued Epstein in the Federal District Court in LA. Assuming the amount in controversy is met, does the court have subject matter jurisdiction?

Yes; the Court has SMJ​ Rose is a citizen of California​ Epstein was physically present in the state of New York and made the intent to indefinitely stay when he cried," I have formed the intent to make this state my permanent home!!!" ​ This occurred prior to the suit being filed​

Holmes Test: does this arise under federal law? Danbury is hired by General Hospital on a one-year contract to work on a medical research project. The funding for the project is provided by the National Institutes of Health, a federal agency. She is fired, allegedly for being late too often. She sues for breach of contract.

there is a vague federal aura to the case, since Danbury was working on a federally funded project. But no federal law creates her cause of action; her suit is for breach of contract. There is no jurisdiction under § 1331.

A wholesaler sued a retailer in a federal court in State A. The retailer timely filed and served a motion to dismiss for lack of subject matter jurisdiction. The court denied this motion. Thereafter, the retailer filed and served his answer. Ten days after serving his answer, the retailer filed an amended answer, raising, for the first time, the defense of lack of personal jurisdiction, which was available when the motion mentioned above was filed. Should the court consider the personal jurisdiction objection? A No, because that defense has been waived. B No, because objections to personal jurisdiction may only be made by making a motion to dismiss before filing an answer. C Yes, because the retailer may serve an amended answer as of right within 21 days after serving his original answer. D Yes, because the amendment relates back to the original answer, thus preserving his right to raise the objection.

•(A) The court should not consider the retailer's objection because the retailer has waived the defense of lack of personal jurisdiction. A defendant may object to personal jurisdiction in two ways: (i) by raising it in a pre-answer motion to dismiss under Rule 12(b); or (ii) if he has not moved under Rule 12(b), by raising the defense in his answer. Thus, the retailer has waived the defense of lack of personal jurisdiction by making a pre-answer motion to dismiss for lack of subject matter jurisdiction under Rule 12(b) and failing to raise in that motion the defense of lack of personal jurisdiction. (B) is incorrect because a defendant may preserve the defense of lack of personal jurisdiction in his answer, provided he has not made a Rule 12(b) motion. (C) is also incorrect. If a defendant does not make a Rule 12(b) motion, he may preserve the defense by raising it in his answer or in any amendment as of right. However, the retailer has made a Rule 12(b) motion to dismiss; therefore, he has not preserved by putting it in his amended answer. (D) is incorrect because, as previously discussed, the retailer waived the defense by making a Rule 12(b) motion. Moreover, the concept of "relation back" is irrelevant here. It is used to determine when a claim asserted in an amended pleading relates back to the date of the original pleading for statute of limitations purposes.


Ensembles d'études connexes

important lecture quizes for final

View Set

AWS Certified Developer Associate Practice Exams 2022 Diagnostic Test

View Set

THE 24-HOUR CLOCK - 24-hour time convert to 12-hour

View Set

PE Units 1-5 Test Questions Review

View Set

Quiz: Chapter 4, Settings for Psychiatric Care

View Set